Dyssynergic defecation

Article Type
Changed
Wed, 04/28/2021 - 13:33

Introduction

About 40% of the population experiences lower GI symptoms suggestive of gastrointestinal motility disorders.1,2 The global prevalence of chronic constipation is 18%, and the condition includes multiple overlapping subtypes.3 Evacuation disorders affect over half (59%) of patients and include dyssynergic defecation (DD).4 The inability to coordinate the abdominal, rectal, pelvic floor, and anal/puborectalis muscles to evacuate stools causes DD.5 The etiology of DD remains unclear and is often misdiagnosed. Clinically, the symptoms of DD overlap with other lower GI disorders, often leading to unnecessary and invasive procedures.2 We describe the clinical characteristics, diagnostic tools, treatment options, and evidence-based approach for the management of DD.

Vidyard Video

Clinical presentation

Over two-thirds of patients with DD acquire this disorder during adulthood, and one-third have symptoms from childhood.6 Though there is not usually an inciting event, 29% of patients report that symptoms began after events such as pregnancy or back injury,6 and opioid users have higher prevalence and severity of DD.7

Dr. Asad Jehangir

Over 80% of patients report excessive straining, feelings of incomplete evacuation, and hard stools, and 50% report sensation of anal blockage or use of digital maneuvers.2 Other symptoms include infrequent bowel movements, abdominal pain, anal pain, and stool leakage.2 Evaluation of DD includes obtaining a detailed history utilizing the Bristol Stool Form Scale;8 however, patients’ recall of stool habit is often inaccurate, which results in suboptimal care.9,10 Prospective stool diaries can help to provide more objective assessment of patients’ symptoms, eliminate recall bias, and provide more reliable information. Several useful questionnaires are available for clinical and research purposes to characterize lower-GI symptoms, including the Constipation Scoring System,11 Patient Assessment of Constipation Symptoms (PAC-SYM),12 and Patient Assessment of Constipation Quality of Life (PAC-QOL).2,13 The Constipation Stool digital app enhances accuracy of data capture and offers a reliable and user-friendly method for recording bowel symptoms for patients, clinicians, and clinical investigators.14

 

Diagnosis

The diagnosis of DD requires careful physical and digital rectal examination together with anorectal manometry and a balloon expulsion test. Defecography and colonic transit studies provide additional assessment.

Dr. Satish S.C. Rao

Physical examination

Abdominal examination should include palpation for stool in the colon and identification of abdominal mass or fecal impaction.2A high-quality digital rectal examination can help to identify patients who could benefit from physiological testing to confirm and treat DD.15 Rectal examination is performed by placing examiner’s lubricated gloved right index finger in a patient’s rectum, with the examiner’s left hand on patient’s abdomen, and asking the patient to push and bear down as if defecating.15 The contraction of the abdominal muscles is felt using the left hand, while the anal sphincter relaxation and degree of perineal descent are felt using the right-hand index finger.15 A diagnosis of dyssynergia is suspected if the digital rectal examination reveals two or more of the following abnormalities: inability to contract abdominal muscles (lack of push effort), inability to relax or paradoxical contraction of the anal sphincter and/or puborectalis, or absence of perineal descent.15 Digital rectal examination has good sensitivity (75%), specificity (87%), and positive predictive value (97%) for DD.16

 

 

High resolution anorectal manometry

Figure 1
Figure - Manometrically there are four different types of DD. (A) The patient with Type I DD is able to generate an adequate propulsive force but with a paradoxical increase in anal sphincter pressure. (B) The patient with Type II DD is unable to generate an adequate propulsive force and paradoxically increases the anal sphincter pressure. (C) The patient with Type III DD is able to generate an adequate propulsive force, with absent or incomplete (< 20%) anal sphincter relaxation. (D) The patient with Type IV DD is unable to generate an adequate propulsive force with absent or incomplete anal sphincter relaxation.

Anorectal manometry (ARM) is the preferred method for the evaluation of defecatory disorders.17,18 ARM is best performed using the high-resolution anorectal manometry (HRAM) systems19 that consist of a flexible probe – 0.5-cm diameter with multiple circumferential sensors along the anal canal – and another two sensors inside a rectal balloon.18 It provides a topographic and waveform display of manometric pressure data (Figure). The 3D high-definition ARM probe is a rigid 1-cm probe that provides 3D topographic profiles.18 ARM is typically performed in both the left lateral position and in a more physiological seated position.20,21 There is considerable variation amongst different institutions on how to perform HRAM, and a recent International Anorectal Physiology Working Group (IAPWG) has provided consensus recommendations for performing this test.22 The procedure for performing HRAM is reviewed elsewhere, but the key elements are summarized below.18

Push maneuver: On HRAM, after the assessment of resting and squeeze anal sphincter pressures, the patient is asked to push or bear down as if to defecate while lying in left lateral decubitus position. The best of two attempts that closely mimics a normal bearing down maneuver is used for categorizing patient’s defecatory pattern.18 In patients with DD, at least four distinct dyssynergia phenotypes have been recognized (Figure),23 though recent studies suggest eight patterns.24 Defecation index (maximum rectal pressure/minimum residual anal pressure when bearing down) greater than 1.2 is considered normal.18

Simulated defecation on commode: The subject is asked to attempt defecation while seated on a commode with intrarectal balloon filled with 60 cc of air, and both the defecation pattern(s) and defecation index are calculated. A lack of coordinated push effort is highly suggestive of DD.21

Rectoanal Inhibitory Reflex (RAIR): RAIR describes the reflex relaxation of the internal anal sphincter after rectal distension. RAIR is dependent on intact autonomic ganglia and myenteric plexus25and is mediated by the release of nitric oxide and vasoactive intestinal peptide.26 The absence of RAIR suggests Hirschsprung disease.22.27.28

Rectal sensory testing: Intermittent balloon distension of the rectum with incremental volumes of air induces a range of rectal sensations that include first sensation, desire to defecate, urgency to defecate, and maximum tolerable volume. Rectal hyposensitivity is diagnosed when two or more sensory thresholds are higher than those seen in normal subjects29.30 and likely results from disruption of afferent gut-brain pathways, cortical perception/rectal wall dysfunction, or both.29 Rectal hyposensitivity affects 40% of patients with constipation30and is associated with DD but not delayed colonic transit.31 Rectal hyposensitivity may also be seen in patients with diabetes or fecal incontinence.18 About two-thirds of patients with rectal hyposensitivity have rectal hypercompliance, and some have megarectum.32 Some patients with DD have coexisting irritable bowel syndrome (IBS) and may have rectal hypersensitivity.18,33 Rectal compliance is measured alongside rectal sensitivity analysis by plotting a graph between the change in intraballoon volume (mL) and change in intrarectal pressures (mm Hg) during incremental balloon distensions.18.34 Rectal hypercompliance may be seen in megarectum and dyssynergic defecation.34,35 Rectal hypocompliance may be seen in patients with inflammatory bowel disease, postpelvic radiation, chronic ischemia, and advanced age.18

Balloon expulsion test: This test is performed by placing a plastic probe with a balloon in the rectum and filling it with 50 cc of warm water. Patients are given 5 minutes to expel the balloon while sitting on a commode. Balloon expulsion time of more than 1 minute suggests a diagnosis of DD,21 although 2 minutes provides a higher level of agreement with manometric findings.36 Balloon type and body position can influence the results.37 Inability to expel the balloon with normal manometric findings is considered an inconclusive finding per the recent London Classification (i.e., it may be associated with generation of anorectal symptoms, but the clinical relevance of this finding is unclear as it may also be seen in healthy subjects).22

 

 

Defecography

Defecography is a dynamic fluoroscopic study performed in the sitting position after injecting 150 mL of barium paste into the patient’s rectum. Defecography provides useful information about structural changes (e.g., rectoceles, enteroceles, rectal prolapse, and intussusception), DD, and descending perineum syndrome.38 Methodological differences, radiation exposure, and poor interobserver agreement have limited its wider use; therefore, anorectal manometry and the balloon expulsion test are recommended for the initial evaluation of DD.39 Magnetic resonance defecography may be more useful.17,38

Colonic transit studies

Colonic transit study can be assessed using radiopaque markers, wireless motility capsule, or scintigraphy. Wireless motility capsule and scintigraphy have the advantage of determining gastric, small bowel, and whole gut transit times as well. About two-thirds of patients with DD have slow transit constipation (STC),6 which improves after treatment of DD.40 Hence, in patients with chronic constipation, evaluation and management of DD is recommended first. If symptoms persist, then consider colonic transit assessment.41 Given the overlapping nature of the conditions, documentation of STC at the outset could facilitate treatment of both.

Diagnostic criteria for DD

Patients should fulfill the following criteria for diagnosis of DD:42,43

  • Fulfill symptom(s) diagnostic criteria for functional constipation and/or constipation-predominant IBS.
  • Demonstrate dyssynergic pattern (Types I-IV; Figure) during attempted defecation on manometry recordings.
  • Meet one or more of the following criteria:
  • Inability to expel an artificial stool (50 mL water-filled balloon) within 1 minute.
  • Inability to evacuate or retention of 50% or more of barium during defecography. (Some institutions use a prolonged colonic transit time: greater than 5 markers or 20% or higher marker retention on a plain abdominal x-Ray at 120 hours after ingestion of one radio-opaque marker capsule containing 24 radio-opaque markers.)

Treatment of DD

The treatment modalities for DD depend on several factors: patient’s age, comorbidities, underlying pathophysiology, and patient expectations. Treatment options include standard management of constipation, but biofeedback therapy is the mainstay.

Standard management

Medications that cause or worsen constipation should be avoided. The patient should consume adequate fluid and exercise regularly. Patients should receive instructions for timed toilet training (twice daily, 30 minutes after meals). Patients should push at about 50%-70% of their ability for no longer than 5 minutes and avoid postponing defecation or use of digital maneuvers to facilitate defecation.42 The patients should take 25 g of soluble fiber (e.g., psyllium) daily. Of note, the benefits of fiber can take days to weeks44 and may be limited in patients with STC and DD.45 Medications including laxatives and intestinal secretagogues (lubiprostone, linaclotide, plecanatide), and enterokinetic agents (prucalopride) can be used as adjunct therapy for management of DD.42 Their use is titrated during and after biofeedback therapy and may decrease after successful treatment.46

Biofeedback therapy

Biofeedback therapy involves operant conditioning techniques using either a solid state anorectal manometry system, electromyography, simulated balloon, or home biofeedback training devices.42,47 The goals of biofeedback therapy are to correct the abdominal pelvic muscle discoordination during defecation and improve rectal sensation to stool if impaired. Biofeedback therapy involves patient education and active training (typically six sessions, 1-2 weeks apart, with each about 30-60 minutes long), followed by a reinforcement stage (three sessions at 3, 6, and 12 months), though there are variations in training protocols.42

Table

The success of biofeedback therapy depends on the patient’s motivation and the therapist’s skills.42 Compared with standard therapy (diet, exercise, pharmacotherapy), biofeedback therapy provides sustained improvement of bowel symptoms and anorectal function. Up to 70%-80% of DD patients show significant improvement of symptoms in randomized controlled trials (Table).48-52 Biofeedback therapy may also improve dyspeptic symptoms.53 Patients with harder stool consistency, greater willingness to participate, lower baseline bowel satisfaction, lower baseline anal sphincter relaxation, and prolonged balloon expulsion time, as well as patients who used digital maneuvers for defection, more commonly respond to biofeedback therapy.54,55 Longstanding laxative use has been associated with decreased response to biofeedback therapy.56 In patients with rectal hyposensitivity, barostat-assisted sensory training is more effective than a hand-held syringe technique.30 In patients with constipation predominant IBS and rectal hyposensitivity, sensory adaption training is more efficacious and better tolerated than escitalopram.30 Biofeedback therapy was afforded a grade A recommendation for treatment of DD by the American and European Societies of Neurogastroenterology and Motility.57

The access to office-based biofeedback therapy may be limited because of costs and low availability. The time required to attend multiple sessions may be burdensome for some patients, especially if they are taking time off from work. A recent study showed that patients with higher level of education may be less likely to adhere to biofeedback therapy.58 Recently, home biofeedback was shown to be noninferior to office biofeedback and was more cost-effective, which provides an alternative option for treating more patients.59

Endoscopic/surgical options

Other less effective treatment options for DD include botulinum toxin injection and myectomy.60-62 Botulinum toxin injection appears to have mixed effects with less than 50% of patients reporting symptomatic improvement, and it may cause fecal incontinence.60,63

 

Conclusion

DD is a common yet poorly recognized cause of constipation. Its clinical presentation overlaps with other lower-GI disorders. Its diagnosis requires detailed history, digital rectal examination, prospective stool diaries, anorectal manometry, and balloon expulsion tests. Biofeedback therapy offers excellent and sustained symptomatic improvement; however, access to office-based biofeedback is limited, and there is an urgent need for home-based biofeedback therapy programs.59

Dr. Rao is J. Harold Harrison Distinguished University Chair, professor of medicine, director of neurogastroenterology/motility, and director of digestive health at the Digestive Health Clinical Research Center Augusta (Georgia) University. He is supported by National Institutes of Health grants R01DK121003-02 and U01DK115572. Dr. Jehangir is a gastroenterology and Hepatology Fellow at the Digestive Health Clinical Research Center at Augusta University. They reported having no conflicts of interest.

 

 

References



1. Peery AF, et al. Gastroenterology. 2012;143(5):1179-1187.e3 .

2. Curtin B, et al. J Neurogastroenterol Motil. 2020 30;26(4):423-36.

3. Suares NC & Ford AC. Am J Gastroenterol. 2011 Sep;106(9):1582-91.

4. Mertz H, et al. Am J Gastroenterol. 1999;94(3):609-15.

5. Rao SS, et al. Am J Gastroenterol. 1998;93(7):1042-50.

6. Rao SSC, et al. J Clin Gastroenterol. 2004;38(8):680-5.

7. Nojkov B, et al. Am J Gastroenterol. 2019;114(11):1772-7.

8. Heaton KW, et al. Gut. 1992;33(6):818-24.

9. Prichard DO & Bharucha AE. 2018 Oct 15;7:F1000 Faculty Rev-1640.

10. Ashraf W, et al. Am J Gastroenterol. 1996;91(1):26-32.

11. Agachan F, et al.. Dis Colon Rectum. 1996;39(6):681-5.

12. Frank L, et al. Scand J Gastroenterol. 1999;34(9):870-7.

13. Marquis P, et al. Scand J Gastroenterol. 2005;40(5):540-51.

14. Yan Y, et al. Gastroenterology. 2020;158(6):S-400.

15. Rao SSC. Am J Gastroenterol. 2018;113(5):635-8.

16. Tantiphlachiva K, et al. Digital rectal examination is a useful tool for identifying patients with dyssynergia. Clin Gastroenterol Hepatol. 2010;8(11):955-60.

17. Carrington EV, et al. Nat Rev Gastroenterol Hepatol. 2018;15(5):309-23.

18. Tetangco EP, et al. Performing and analyzing high-resolution anorectal manometry. NeuroGastroLatam Rev. 2018;2:120-32.

19. Lee YY, et al. Curr Gastroenterol Rep. 2013;15(12):360.

20. Sharma M, et al. Neurogastroenterol Motil. 2020;32(10):e13910.

21. Rao SSC, et al.. Am J Gastroenterol. 2006;101(12):2790-6.

22. Carrington EV, et al. Neurogastroenterol Motil. 2020;32(1):e13679.

23. Rao SSC. Gastroenterol Clin North Am. 2008;37(3):569-86, viii.

24. Rao SSC, et al. Gastroenterology. 2016;150(4):S158-9.

25. Guinet A, et al. Int J Colorectal Dis. 2011;26(4):507-13.

26. Rattan S, et al. Gastroenterology. 1992;103(1):43-50.

27. Remes-Troche JM & Rao SSC. 2008;2(3):323-35.

28. Zaafouri H, et al..Int J Surgery. 2015. 2(1):9-17.

29. Remes-Troche JM, et al. Dis Colon Rectum. 2010;53(7):1047-54.

 

 

30. Rao SSC, et al. Gastroenterology. 2013;144(5):S-363.

31. Yu T, et al. Medicine (Baltimore). 2016;95(19):e3667.

32. Gladman MA, et al. Neurogastroenterol Motil. 2009;21(5):508-16, e4-5.

33. Lee KJ, et al. Digestion. 2006;73(2-3):133-41 .

34. Rao SSC, et al. Neurogastroenterol Motil. 2002;14(5):553-9.

35. Coss-Adame E, et al.. Clin Gastroenterol Hepatol. 2015;13(6):1143-1150.e1.

36. Chiarioni G, et al. Clin Gastroenterol Hepatol. 2014;12(12):2049-54.

37. Gu G, et al. Gastroenterology. 2018;154(6):S-545–S-546.

38. Savoye-Collet C, et al.. Gastroenterol Clin North Am. 2008;37(3):553-67, viii.

39. Videlock EJ, et al. Neurogastroenterol Motil. 2013;25(6):509-20.

40. Rao SSC, et al. Neurogastroenterol Motil. 2004;16(5):589-96.

41. Wald A, et al. Am J Gastroenterol. 2014;109(8):1141-57 ; (Quiz) 1058.

42. Rao SSC & Patcharatrakul T. J Neurogastroenterol Motil. 2016;22(3):423-35.

43. Rao SS, et al. Functional Anorectal Disorders. Gastroenterology. 2016. S0016-5085(16)00175-X.

44. Bharucha AE, et al.. Gastroenterology. 2013;144(1):218-38.

45. Voderholzer WA, et al. Am J Gastroenterol. 1997;92(1):95-8.

46. Lee HJ, et al. Neurogastroenterol Motil. 2015;27(6):787-95.

47. Simón MA & Bueno AM. J Clin Gastroenterol. 2017;51(10):e90-4.

48. Chiarioni G,et al.. Gastroenterology. 2006;130(3):657-64.

49. Heymen S, et al.. Dis Colon Rectum. 2007;50(4):428-41.

50. Rao SSC, et al. Clin Gastroenterol Hepatol. 2007;5(3):331-8.

51. Rao SSC, et al. Am J Gastroenterol. 2010;105(4):890-6.

52. Patcharatrakul T, et al. Biofeedback therapy. In Clinical and basic neurogastroenterology and motility. India: Stacy Masucci; 2020:517-32.

53. Huaman J-W, et al. Clin Gastroenterol Hepatol. 2020;18(11):2463-2470.e1.

54. Patcharatrakul T, et al. Clin Gastroenterol Hepatol. 2018;16(5):715-21.

55. Chaudhry A, et al. Gastroenterology. 2020;158(6):S-382–S-383.

56. Shim LSE, et al. Aliment Pharmacol Ther. 2011;33(11):1245-51.

57. Rao SSC, et al. Neurogastroenterol Motil. 2015;27(5):594-609.

58. Jangsirikul S, et al. Gastroenterology. 2020;158(6):S-383.

59. Rao SSC, et al. Am J Gastroenterol. 2019;114(6):938-44.

60. Ron Y, et al.. Dis Colon Rectum. 2001;44(12):1821-6.

61. Podzemny V, et al. World J Gastroenterol. 2015;21(4):1053-60.

62. Faried M, et al. J Gastrointest Surg. 2010;14(8):1235-43.

63. Hallan RI, et al. Lancet. 1988;2(8613):714-7.

Publications
Topics
Sections

Introduction

About 40% of the population experiences lower GI symptoms suggestive of gastrointestinal motility disorders.1,2 The global prevalence of chronic constipation is 18%, and the condition includes multiple overlapping subtypes.3 Evacuation disorders affect over half (59%) of patients and include dyssynergic defecation (DD).4 The inability to coordinate the abdominal, rectal, pelvic floor, and anal/puborectalis muscles to evacuate stools causes DD.5 The etiology of DD remains unclear and is often misdiagnosed. Clinically, the symptoms of DD overlap with other lower GI disorders, often leading to unnecessary and invasive procedures.2 We describe the clinical characteristics, diagnostic tools, treatment options, and evidence-based approach for the management of DD.

Vidyard Video

Clinical presentation

Over two-thirds of patients with DD acquire this disorder during adulthood, and one-third have symptoms from childhood.6 Though there is not usually an inciting event, 29% of patients report that symptoms began after events such as pregnancy or back injury,6 and opioid users have higher prevalence and severity of DD.7

Dr. Asad Jehangir

Over 80% of patients report excessive straining, feelings of incomplete evacuation, and hard stools, and 50% report sensation of anal blockage or use of digital maneuvers.2 Other symptoms include infrequent bowel movements, abdominal pain, anal pain, and stool leakage.2 Evaluation of DD includes obtaining a detailed history utilizing the Bristol Stool Form Scale;8 however, patients’ recall of stool habit is often inaccurate, which results in suboptimal care.9,10 Prospective stool diaries can help to provide more objective assessment of patients’ symptoms, eliminate recall bias, and provide more reliable information. Several useful questionnaires are available for clinical and research purposes to characterize lower-GI symptoms, including the Constipation Scoring System,11 Patient Assessment of Constipation Symptoms (PAC-SYM),12 and Patient Assessment of Constipation Quality of Life (PAC-QOL).2,13 The Constipation Stool digital app enhances accuracy of data capture and offers a reliable and user-friendly method for recording bowel symptoms for patients, clinicians, and clinical investigators.14

 

Diagnosis

The diagnosis of DD requires careful physical and digital rectal examination together with anorectal manometry and a balloon expulsion test. Defecography and colonic transit studies provide additional assessment.

Dr. Satish S.C. Rao

Physical examination

Abdominal examination should include palpation for stool in the colon and identification of abdominal mass or fecal impaction.2A high-quality digital rectal examination can help to identify patients who could benefit from physiological testing to confirm and treat DD.15 Rectal examination is performed by placing examiner’s lubricated gloved right index finger in a patient’s rectum, with the examiner’s left hand on patient’s abdomen, and asking the patient to push and bear down as if defecating.15 The contraction of the abdominal muscles is felt using the left hand, while the anal sphincter relaxation and degree of perineal descent are felt using the right-hand index finger.15 A diagnosis of dyssynergia is suspected if the digital rectal examination reveals two or more of the following abnormalities: inability to contract abdominal muscles (lack of push effort), inability to relax or paradoxical contraction of the anal sphincter and/or puborectalis, or absence of perineal descent.15 Digital rectal examination has good sensitivity (75%), specificity (87%), and positive predictive value (97%) for DD.16

 

 

High resolution anorectal manometry

Figure 1
Figure - Manometrically there are four different types of DD. (A) The patient with Type I DD is able to generate an adequate propulsive force but with a paradoxical increase in anal sphincter pressure. (B) The patient with Type II DD is unable to generate an adequate propulsive force and paradoxically increases the anal sphincter pressure. (C) The patient with Type III DD is able to generate an adequate propulsive force, with absent or incomplete (< 20%) anal sphincter relaxation. (D) The patient with Type IV DD is unable to generate an adequate propulsive force with absent or incomplete anal sphincter relaxation.

Anorectal manometry (ARM) is the preferred method for the evaluation of defecatory disorders.17,18 ARM is best performed using the high-resolution anorectal manometry (HRAM) systems19 that consist of a flexible probe – 0.5-cm diameter with multiple circumferential sensors along the anal canal – and another two sensors inside a rectal balloon.18 It provides a topographic and waveform display of manometric pressure data (Figure). The 3D high-definition ARM probe is a rigid 1-cm probe that provides 3D topographic profiles.18 ARM is typically performed in both the left lateral position and in a more physiological seated position.20,21 There is considerable variation amongst different institutions on how to perform HRAM, and a recent International Anorectal Physiology Working Group (IAPWG) has provided consensus recommendations for performing this test.22 The procedure for performing HRAM is reviewed elsewhere, but the key elements are summarized below.18

Push maneuver: On HRAM, after the assessment of resting and squeeze anal sphincter pressures, the patient is asked to push or bear down as if to defecate while lying in left lateral decubitus position. The best of two attempts that closely mimics a normal bearing down maneuver is used for categorizing patient’s defecatory pattern.18 In patients with DD, at least four distinct dyssynergia phenotypes have been recognized (Figure),23 though recent studies suggest eight patterns.24 Defecation index (maximum rectal pressure/minimum residual anal pressure when bearing down) greater than 1.2 is considered normal.18

Simulated defecation on commode: The subject is asked to attempt defecation while seated on a commode with intrarectal balloon filled with 60 cc of air, and both the defecation pattern(s) and defecation index are calculated. A lack of coordinated push effort is highly suggestive of DD.21

Rectoanal Inhibitory Reflex (RAIR): RAIR describes the reflex relaxation of the internal anal sphincter after rectal distension. RAIR is dependent on intact autonomic ganglia and myenteric plexus25and is mediated by the release of nitric oxide and vasoactive intestinal peptide.26 The absence of RAIR suggests Hirschsprung disease.22.27.28

Rectal sensory testing: Intermittent balloon distension of the rectum with incremental volumes of air induces a range of rectal sensations that include first sensation, desire to defecate, urgency to defecate, and maximum tolerable volume. Rectal hyposensitivity is diagnosed when two or more sensory thresholds are higher than those seen in normal subjects29.30 and likely results from disruption of afferent gut-brain pathways, cortical perception/rectal wall dysfunction, or both.29 Rectal hyposensitivity affects 40% of patients with constipation30and is associated with DD but not delayed colonic transit.31 Rectal hyposensitivity may also be seen in patients with diabetes or fecal incontinence.18 About two-thirds of patients with rectal hyposensitivity have rectal hypercompliance, and some have megarectum.32 Some patients with DD have coexisting irritable bowel syndrome (IBS) and may have rectal hypersensitivity.18,33 Rectal compliance is measured alongside rectal sensitivity analysis by plotting a graph between the change in intraballoon volume (mL) and change in intrarectal pressures (mm Hg) during incremental balloon distensions.18.34 Rectal hypercompliance may be seen in megarectum and dyssynergic defecation.34,35 Rectal hypocompliance may be seen in patients with inflammatory bowel disease, postpelvic radiation, chronic ischemia, and advanced age.18

Balloon expulsion test: This test is performed by placing a plastic probe with a balloon in the rectum and filling it with 50 cc of warm water. Patients are given 5 minutes to expel the balloon while sitting on a commode. Balloon expulsion time of more than 1 minute suggests a diagnosis of DD,21 although 2 minutes provides a higher level of agreement with manometric findings.36 Balloon type and body position can influence the results.37 Inability to expel the balloon with normal manometric findings is considered an inconclusive finding per the recent London Classification (i.e., it may be associated with generation of anorectal symptoms, but the clinical relevance of this finding is unclear as it may also be seen in healthy subjects).22

 

 

Defecography

Defecography is a dynamic fluoroscopic study performed in the sitting position after injecting 150 mL of barium paste into the patient’s rectum. Defecography provides useful information about structural changes (e.g., rectoceles, enteroceles, rectal prolapse, and intussusception), DD, and descending perineum syndrome.38 Methodological differences, radiation exposure, and poor interobserver agreement have limited its wider use; therefore, anorectal manometry and the balloon expulsion test are recommended for the initial evaluation of DD.39 Magnetic resonance defecography may be more useful.17,38

Colonic transit studies

Colonic transit study can be assessed using radiopaque markers, wireless motility capsule, or scintigraphy. Wireless motility capsule and scintigraphy have the advantage of determining gastric, small bowel, and whole gut transit times as well. About two-thirds of patients with DD have slow transit constipation (STC),6 which improves after treatment of DD.40 Hence, in patients with chronic constipation, evaluation and management of DD is recommended first. If symptoms persist, then consider colonic transit assessment.41 Given the overlapping nature of the conditions, documentation of STC at the outset could facilitate treatment of both.

Diagnostic criteria for DD

Patients should fulfill the following criteria for diagnosis of DD:42,43

  • Fulfill symptom(s) diagnostic criteria for functional constipation and/or constipation-predominant IBS.
  • Demonstrate dyssynergic pattern (Types I-IV; Figure) during attempted defecation on manometry recordings.
  • Meet one or more of the following criteria:
  • Inability to expel an artificial stool (50 mL water-filled balloon) within 1 minute.
  • Inability to evacuate or retention of 50% or more of barium during defecography. (Some institutions use a prolonged colonic transit time: greater than 5 markers or 20% or higher marker retention on a plain abdominal x-Ray at 120 hours after ingestion of one radio-opaque marker capsule containing 24 radio-opaque markers.)

Treatment of DD

The treatment modalities for DD depend on several factors: patient’s age, comorbidities, underlying pathophysiology, and patient expectations. Treatment options include standard management of constipation, but biofeedback therapy is the mainstay.

Standard management

Medications that cause or worsen constipation should be avoided. The patient should consume adequate fluid and exercise regularly. Patients should receive instructions for timed toilet training (twice daily, 30 minutes after meals). Patients should push at about 50%-70% of their ability for no longer than 5 minutes and avoid postponing defecation or use of digital maneuvers to facilitate defecation.42 The patients should take 25 g of soluble fiber (e.g., psyllium) daily. Of note, the benefits of fiber can take days to weeks44 and may be limited in patients with STC and DD.45 Medications including laxatives and intestinal secretagogues (lubiprostone, linaclotide, plecanatide), and enterokinetic agents (prucalopride) can be used as adjunct therapy for management of DD.42 Their use is titrated during and after biofeedback therapy and may decrease after successful treatment.46

Biofeedback therapy

Biofeedback therapy involves operant conditioning techniques using either a solid state anorectal manometry system, electromyography, simulated balloon, or home biofeedback training devices.42,47 The goals of biofeedback therapy are to correct the abdominal pelvic muscle discoordination during defecation and improve rectal sensation to stool if impaired. Biofeedback therapy involves patient education and active training (typically six sessions, 1-2 weeks apart, with each about 30-60 minutes long), followed by a reinforcement stage (three sessions at 3, 6, and 12 months), though there are variations in training protocols.42

Table

The success of biofeedback therapy depends on the patient’s motivation and the therapist’s skills.42 Compared with standard therapy (diet, exercise, pharmacotherapy), biofeedback therapy provides sustained improvement of bowel symptoms and anorectal function. Up to 70%-80% of DD patients show significant improvement of symptoms in randomized controlled trials (Table).48-52 Biofeedback therapy may also improve dyspeptic symptoms.53 Patients with harder stool consistency, greater willingness to participate, lower baseline bowel satisfaction, lower baseline anal sphincter relaxation, and prolonged balloon expulsion time, as well as patients who used digital maneuvers for defection, more commonly respond to biofeedback therapy.54,55 Longstanding laxative use has been associated with decreased response to biofeedback therapy.56 In patients with rectal hyposensitivity, barostat-assisted sensory training is more effective than a hand-held syringe technique.30 In patients with constipation predominant IBS and rectal hyposensitivity, sensory adaption training is more efficacious and better tolerated than escitalopram.30 Biofeedback therapy was afforded a grade A recommendation for treatment of DD by the American and European Societies of Neurogastroenterology and Motility.57

The access to office-based biofeedback therapy may be limited because of costs and low availability. The time required to attend multiple sessions may be burdensome for some patients, especially if they are taking time off from work. A recent study showed that patients with higher level of education may be less likely to adhere to biofeedback therapy.58 Recently, home biofeedback was shown to be noninferior to office biofeedback and was more cost-effective, which provides an alternative option for treating more patients.59

Endoscopic/surgical options

Other less effective treatment options for DD include botulinum toxin injection and myectomy.60-62 Botulinum toxin injection appears to have mixed effects with less than 50% of patients reporting symptomatic improvement, and it may cause fecal incontinence.60,63

 

Conclusion

DD is a common yet poorly recognized cause of constipation. Its clinical presentation overlaps with other lower-GI disorders. Its diagnosis requires detailed history, digital rectal examination, prospective stool diaries, anorectal manometry, and balloon expulsion tests. Biofeedback therapy offers excellent and sustained symptomatic improvement; however, access to office-based biofeedback is limited, and there is an urgent need for home-based biofeedback therapy programs.59

Dr. Rao is J. Harold Harrison Distinguished University Chair, professor of medicine, director of neurogastroenterology/motility, and director of digestive health at the Digestive Health Clinical Research Center Augusta (Georgia) University. He is supported by National Institutes of Health grants R01DK121003-02 and U01DK115572. Dr. Jehangir is a gastroenterology and Hepatology Fellow at the Digestive Health Clinical Research Center at Augusta University. They reported having no conflicts of interest.

 

 

References



1. Peery AF, et al. Gastroenterology. 2012;143(5):1179-1187.e3 .

2. Curtin B, et al. J Neurogastroenterol Motil. 2020 30;26(4):423-36.

3. Suares NC & Ford AC. Am J Gastroenterol. 2011 Sep;106(9):1582-91.

4. Mertz H, et al. Am J Gastroenterol. 1999;94(3):609-15.

5. Rao SS, et al. Am J Gastroenterol. 1998;93(7):1042-50.

6. Rao SSC, et al. J Clin Gastroenterol. 2004;38(8):680-5.

7. Nojkov B, et al. Am J Gastroenterol. 2019;114(11):1772-7.

8. Heaton KW, et al. Gut. 1992;33(6):818-24.

9. Prichard DO & Bharucha AE. 2018 Oct 15;7:F1000 Faculty Rev-1640.

10. Ashraf W, et al. Am J Gastroenterol. 1996;91(1):26-32.

11. Agachan F, et al.. Dis Colon Rectum. 1996;39(6):681-5.

12. Frank L, et al. Scand J Gastroenterol. 1999;34(9):870-7.

13. Marquis P, et al. Scand J Gastroenterol. 2005;40(5):540-51.

14. Yan Y, et al. Gastroenterology. 2020;158(6):S-400.

15. Rao SSC. Am J Gastroenterol. 2018;113(5):635-8.

16. Tantiphlachiva K, et al. Digital rectal examination is a useful tool for identifying patients with dyssynergia. Clin Gastroenterol Hepatol. 2010;8(11):955-60.

17. Carrington EV, et al. Nat Rev Gastroenterol Hepatol. 2018;15(5):309-23.

18. Tetangco EP, et al. Performing and analyzing high-resolution anorectal manometry. NeuroGastroLatam Rev. 2018;2:120-32.

19. Lee YY, et al. Curr Gastroenterol Rep. 2013;15(12):360.

20. Sharma M, et al. Neurogastroenterol Motil. 2020;32(10):e13910.

21. Rao SSC, et al.. Am J Gastroenterol. 2006;101(12):2790-6.

22. Carrington EV, et al. Neurogastroenterol Motil. 2020;32(1):e13679.

23. Rao SSC. Gastroenterol Clin North Am. 2008;37(3):569-86, viii.

24. Rao SSC, et al. Gastroenterology. 2016;150(4):S158-9.

25. Guinet A, et al. Int J Colorectal Dis. 2011;26(4):507-13.

26. Rattan S, et al. Gastroenterology. 1992;103(1):43-50.

27. Remes-Troche JM & Rao SSC. 2008;2(3):323-35.

28. Zaafouri H, et al..Int J Surgery. 2015. 2(1):9-17.

29. Remes-Troche JM, et al. Dis Colon Rectum. 2010;53(7):1047-54.

 

 

30. Rao SSC, et al. Gastroenterology. 2013;144(5):S-363.

31. Yu T, et al. Medicine (Baltimore). 2016;95(19):e3667.

32. Gladman MA, et al. Neurogastroenterol Motil. 2009;21(5):508-16, e4-5.

33. Lee KJ, et al. Digestion. 2006;73(2-3):133-41 .

34. Rao SSC, et al. Neurogastroenterol Motil. 2002;14(5):553-9.

35. Coss-Adame E, et al.. Clin Gastroenterol Hepatol. 2015;13(6):1143-1150.e1.

36. Chiarioni G, et al. Clin Gastroenterol Hepatol. 2014;12(12):2049-54.

37. Gu G, et al. Gastroenterology. 2018;154(6):S-545–S-546.

38. Savoye-Collet C, et al.. Gastroenterol Clin North Am. 2008;37(3):553-67, viii.

39. Videlock EJ, et al. Neurogastroenterol Motil. 2013;25(6):509-20.

40. Rao SSC, et al. Neurogastroenterol Motil. 2004;16(5):589-96.

41. Wald A, et al. Am J Gastroenterol. 2014;109(8):1141-57 ; (Quiz) 1058.

42. Rao SSC & Patcharatrakul T. J Neurogastroenterol Motil. 2016;22(3):423-35.

43. Rao SS, et al. Functional Anorectal Disorders. Gastroenterology. 2016. S0016-5085(16)00175-X.

44. Bharucha AE, et al.. Gastroenterology. 2013;144(1):218-38.

45. Voderholzer WA, et al. Am J Gastroenterol. 1997;92(1):95-8.

46. Lee HJ, et al. Neurogastroenterol Motil. 2015;27(6):787-95.

47. Simón MA & Bueno AM. J Clin Gastroenterol. 2017;51(10):e90-4.

48. Chiarioni G,et al.. Gastroenterology. 2006;130(3):657-64.

49. Heymen S, et al.. Dis Colon Rectum. 2007;50(4):428-41.

50. Rao SSC, et al. Clin Gastroenterol Hepatol. 2007;5(3):331-8.

51. Rao SSC, et al. Am J Gastroenterol. 2010;105(4):890-6.

52. Patcharatrakul T, et al. Biofeedback therapy. In Clinical and basic neurogastroenterology and motility. India: Stacy Masucci; 2020:517-32.

53. Huaman J-W, et al. Clin Gastroenterol Hepatol. 2020;18(11):2463-2470.e1.

54. Patcharatrakul T, et al. Clin Gastroenterol Hepatol. 2018;16(5):715-21.

55. Chaudhry A, et al. Gastroenterology. 2020;158(6):S-382–S-383.

56. Shim LSE, et al. Aliment Pharmacol Ther. 2011;33(11):1245-51.

57. Rao SSC, et al. Neurogastroenterol Motil. 2015;27(5):594-609.

58. Jangsirikul S, et al. Gastroenterology. 2020;158(6):S-383.

59. Rao SSC, et al. Am J Gastroenterol. 2019;114(6):938-44.

60. Ron Y, et al.. Dis Colon Rectum. 2001;44(12):1821-6.

61. Podzemny V, et al. World J Gastroenterol. 2015;21(4):1053-60.

62. Faried M, et al. J Gastrointest Surg. 2010;14(8):1235-43.

63. Hallan RI, et al. Lancet. 1988;2(8613):714-7.

Introduction

About 40% of the population experiences lower GI symptoms suggestive of gastrointestinal motility disorders.1,2 The global prevalence of chronic constipation is 18%, and the condition includes multiple overlapping subtypes.3 Evacuation disorders affect over half (59%) of patients and include dyssynergic defecation (DD).4 The inability to coordinate the abdominal, rectal, pelvic floor, and anal/puborectalis muscles to evacuate stools causes DD.5 The etiology of DD remains unclear and is often misdiagnosed. Clinically, the symptoms of DD overlap with other lower GI disorders, often leading to unnecessary and invasive procedures.2 We describe the clinical characteristics, diagnostic tools, treatment options, and evidence-based approach for the management of DD.

Vidyard Video

Clinical presentation

Over two-thirds of patients with DD acquire this disorder during adulthood, and one-third have symptoms from childhood.6 Though there is not usually an inciting event, 29% of patients report that symptoms began after events such as pregnancy or back injury,6 and opioid users have higher prevalence and severity of DD.7

Dr. Asad Jehangir

Over 80% of patients report excessive straining, feelings of incomplete evacuation, and hard stools, and 50% report sensation of anal blockage or use of digital maneuvers.2 Other symptoms include infrequent bowel movements, abdominal pain, anal pain, and stool leakage.2 Evaluation of DD includes obtaining a detailed history utilizing the Bristol Stool Form Scale;8 however, patients’ recall of stool habit is often inaccurate, which results in suboptimal care.9,10 Prospective stool diaries can help to provide more objective assessment of patients’ symptoms, eliminate recall bias, and provide more reliable information. Several useful questionnaires are available for clinical and research purposes to characterize lower-GI symptoms, including the Constipation Scoring System,11 Patient Assessment of Constipation Symptoms (PAC-SYM),12 and Patient Assessment of Constipation Quality of Life (PAC-QOL).2,13 The Constipation Stool digital app enhances accuracy of data capture and offers a reliable and user-friendly method for recording bowel symptoms for patients, clinicians, and clinical investigators.14

 

Diagnosis

The diagnosis of DD requires careful physical and digital rectal examination together with anorectal manometry and a balloon expulsion test. Defecography and colonic transit studies provide additional assessment.

Dr. Satish S.C. Rao

Physical examination

Abdominal examination should include palpation for stool in the colon and identification of abdominal mass or fecal impaction.2A high-quality digital rectal examination can help to identify patients who could benefit from physiological testing to confirm and treat DD.15 Rectal examination is performed by placing examiner’s lubricated gloved right index finger in a patient’s rectum, with the examiner’s left hand on patient’s abdomen, and asking the patient to push and bear down as if defecating.15 The contraction of the abdominal muscles is felt using the left hand, while the anal sphincter relaxation and degree of perineal descent are felt using the right-hand index finger.15 A diagnosis of dyssynergia is suspected if the digital rectal examination reveals two or more of the following abnormalities: inability to contract abdominal muscles (lack of push effort), inability to relax or paradoxical contraction of the anal sphincter and/or puborectalis, or absence of perineal descent.15 Digital rectal examination has good sensitivity (75%), specificity (87%), and positive predictive value (97%) for DD.16

 

 

High resolution anorectal manometry

Figure 1
Figure - Manometrically there are four different types of DD. (A) The patient with Type I DD is able to generate an adequate propulsive force but with a paradoxical increase in anal sphincter pressure. (B) The patient with Type II DD is unable to generate an adequate propulsive force and paradoxically increases the anal sphincter pressure. (C) The patient with Type III DD is able to generate an adequate propulsive force, with absent or incomplete (< 20%) anal sphincter relaxation. (D) The patient with Type IV DD is unable to generate an adequate propulsive force with absent or incomplete anal sphincter relaxation.

Anorectal manometry (ARM) is the preferred method for the evaluation of defecatory disorders.17,18 ARM is best performed using the high-resolution anorectal manometry (HRAM) systems19 that consist of a flexible probe – 0.5-cm diameter with multiple circumferential sensors along the anal canal – and another two sensors inside a rectal balloon.18 It provides a topographic and waveform display of manometric pressure data (Figure). The 3D high-definition ARM probe is a rigid 1-cm probe that provides 3D topographic profiles.18 ARM is typically performed in both the left lateral position and in a more physiological seated position.20,21 There is considerable variation amongst different institutions on how to perform HRAM, and a recent International Anorectal Physiology Working Group (IAPWG) has provided consensus recommendations for performing this test.22 The procedure for performing HRAM is reviewed elsewhere, but the key elements are summarized below.18

Push maneuver: On HRAM, after the assessment of resting and squeeze anal sphincter pressures, the patient is asked to push or bear down as if to defecate while lying in left lateral decubitus position. The best of two attempts that closely mimics a normal bearing down maneuver is used for categorizing patient’s defecatory pattern.18 In patients with DD, at least four distinct dyssynergia phenotypes have been recognized (Figure),23 though recent studies suggest eight patterns.24 Defecation index (maximum rectal pressure/minimum residual anal pressure when bearing down) greater than 1.2 is considered normal.18

Simulated defecation on commode: The subject is asked to attempt defecation while seated on a commode with intrarectal balloon filled with 60 cc of air, and both the defecation pattern(s) and defecation index are calculated. A lack of coordinated push effort is highly suggestive of DD.21

Rectoanal Inhibitory Reflex (RAIR): RAIR describes the reflex relaxation of the internal anal sphincter after rectal distension. RAIR is dependent on intact autonomic ganglia and myenteric plexus25and is mediated by the release of nitric oxide and vasoactive intestinal peptide.26 The absence of RAIR suggests Hirschsprung disease.22.27.28

Rectal sensory testing: Intermittent balloon distension of the rectum with incremental volumes of air induces a range of rectal sensations that include first sensation, desire to defecate, urgency to defecate, and maximum tolerable volume. Rectal hyposensitivity is diagnosed when two or more sensory thresholds are higher than those seen in normal subjects29.30 and likely results from disruption of afferent gut-brain pathways, cortical perception/rectal wall dysfunction, or both.29 Rectal hyposensitivity affects 40% of patients with constipation30and is associated with DD but not delayed colonic transit.31 Rectal hyposensitivity may also be seen in patients with diabetes or fecal incontinence.18 About two-thirds of patients with rectal hyposensitivity have rectal hypercompliance, and some have megarectum.32 Some patients with DD have coexisting irritable bowel syndrome (IBS) and may have rectal hypersensitivity.18,33 Rectal compliance is measured alongside rectal sensitivity analysis by plotting a graph between the change in intraballoon volume (mL) and change in intrarectal pressures (mm Hg) during incremental balloon distensions.18.34 Rectal hypercompliance may be seen in megarectum and dyssynergic defecation.34,35 Rectal hypocompliance may be seen in patients with inflammatory bowel disease, postpelvic radiation, chronic ischemia, and advanced age.18

Balloon expulsion test: This test is performed by placing a plastic probe with a balloon in the rectum and filling it with 50 cc of warm water. Patients are given 5 minutes to expel the balloon while sitting on a commode. Balloon expulsion time of more than 1 minute suggests a diagnosis of DD,21 although 2 minutes provides a higher level of agreement with manometric findings.36 Balloon type and body position can influence the results.37 Inability to expel the balloon with normal manometric findings is considered an inconclusive finding per the recent London Classification (i.e., it may be associated with generation of anorectal symptoms, but the clinical relevance of this finding is unclear as it may also be seen in healthy subjects).22

 

 

Defecography

Defecography is a dynamic fluoroscopic study performed in the sitting position after injecting 150 mL of barium paste into the patient’s rectum. Defecography provides useful information about structural changes (e.g., rectoceles, enteroceles, rectal prolapse, and intussusception), DD, and descending perineum syndrome.38 Methodological differences, radiation exposure, and poor interobserver agreement have limited its wider use; therefore, anorectal manometry and the balloon expulsion test are recommended for the initial evaluation of DD.39 Magnetic resonance defecography may be more useful.17,38

Colonic transit studies

Colonic transit study can be assessed using radiopaque markers, wireless motility capsule, or scintigraphy. Wireless motility capsule and scintigraphy have the advantage of determining gastric, small bowel, and whole gut transit times as well. About two-thirds of patients with DD have slow transit constipation (STC),6 which improves after treatment of DD.40 Hence, in patients with chronic constipation, evaluation and management of DD is recommended first. If symptoms persist, then consider colonic transit assessment.41 Given the overlapping nature of the conditions, documentation of STC at the outset could facilitate treatment of both.

Diagnostic criteria for DD

Patients should fulfill the following criteria for diagnosis of DD:42,43

  • Fulfill symptom(s) diagnostic criteria for functional constipation and/or constipation-predominant IBS.
  • Demonstrate dyssynergic pattern (Types I-IV; Figure) during attempted defecation on manometry recordings.
  • Meet one or more of the following criteria:
  • Inability to expel an artificial stool (50 mL water-filled balloon) within 1 minute.
  • Inability to evacuate or retention of 50% or more of barium during defecography. (Some institutions use a prolonged colonic transit time: greater than 5 markers or 20% or higher marker retention on a plain abdominal x-Ray at 120 hours after ingestion of one radio-opaque marker capsule containing 24 radio-opaque markers.)

Treatment of DD

The treatment modalities for DD depend on several factors: patient’s age, comorbidities, underlying pathophysiology, and patient expectations. Treatment options include standard management of constipation, but biofeedback therapy is the mainstay.

Standard management

Medications that cause or worsen constipation should be avoided. The patient should consume adequate fluid and exercise regularly. Patients should receive instructions for timed toilet training (twice daily, 30 minutes after meals). Patients should push at about 50%-70% of their ability for no longer than 5 minutes and avoid postponing defecation or use of digital maneuvers to facilitate defecation.42 The patients should take 25 g of soluble fiber (e.g., psyllium) daily. Of note, the benefits of fiber can take days to weeks44 and may be limited in patients with STC and DD.45 Medications including laxatives and intestinal secretagogues (lubiprostone, linaclotide, plecanatide), and enterokinetic agents (prucalopride) can be used as adjunct therapy for management of DD.42 Their use is titrated during and after biofeedback therapy and may decrease after successful treatment.46

Biofeedback therapy

Biofeedback therapy involves operant conditioning techniques using either a solid state anorectal manometry system, electromyography, simulated balloon, or home biofeedback training devices.42,47 The goals of biofeedback therapy are to correct the abdominal pelvic muscle discoordination during defecation and improve rectal sensation to stool if impaired. Biofeedback therapy involves patient education and active training (typically six sessions, 1-2 weeks apart, with each about 30-60 minutes long), followed by a reinforcement stage (three sessions at 3, 6, and 12 months), though there are variations in training protocols.42

Table

The success of biofeedback therapy depends on the patient’s motivation and the therapist’s skills.42 Compared with standard therapy (diet, exercise, pharmacotherapy), biofeedback therapy provides sustained improvement of bowel symptoms and anorectal function. Up to 70%-80% of DD patients show significant improvement of symptoms in randomized controlled trials (Table).48-52 Biofeedback therapy may also improve dyspeptic symptoms.53 Patients with harder stool consistency, greater willingness to participate, lower baseline bowel satisfaction, lower baseline anal sphincter relaxation, and prolonged balloon expulsion time, as well as patients who used digital maneuvers for defection, more commonly respond to biofeedback therapy.54,55 Longstanding laxative use has been associated with decreased response to biofeedback therapy.56 In patients with rectal hyposensitivity, barostat-assisted sensory training is more effective than a hand-held syringe technique.30 In patients with constipation predominant IBS and rectal hyposensitivity, sensory adaption training is more efficacious and better tolerated than escitalopram.30 Biofeedback therapy was afforded a grade A recommendation for treatment of DD by the American and European Societies of Neurogastroenterology and Motility.57

The access to office-based biofeedback therapy may be limited because of costs and low availability. The time required to attend multiple sessions may be burdensome for some patients, especially if they are taking time off from work. A recent study showed that patients with higher level of education may be less likely to adhere to biofeedback therapy.58 Recently, home biofeedback was shown to be noninferior to office biofeedback and was more cost-effective, which provides an alternative option for treating more patients.59

Endoscopic/surgical options

Other less effective treatment options for DD include botulinum toxin injection and myectomy.60-62 Botulinum toxin injection appears to have mixed effects with less than 50% of patients reporting symptomatic improvement, and it may cause fecal incontinence.60,63

 

Conclusion

DD is a common yet poorly recognized cause of constipation. Its clinical presentation overlaps with other lower-GI disorders. Its diagnosis requires detailed history, digital rectal examination, prospective stool diaries, anorectal manometry, and balloon expulsion tests. Biofeedback therapy offers excellent and sustained symptomatic improvement; however, access to office-based biofeedback is limited, and there is an urgent need for home-based biofeedback therapy programs.59

Dr. Rao is J. Harold Harrison Distinguished University Chair, professor of medicine, director of neurogastroenterology/motility, and director of digestive health at the Digestive Health Clinical Research Center Augusta (Georgia) University. He is supported by National Institutes of Health grants R01DK121003-02 and U01DK115572. Dr. Jehangir is a gastroenterology and Hepatology Fellow at the Digestive Health Clinical Research Center at Augusta University. They reported having no conflicts of interest.

 

 

References



1. Peery AF, et al. Gastroenterology. 2012;143(5):1179-1187.e3 .

2. Curtin B, et al. J Neurogastroenterol Motil. 2020 30;26(4):423-36.

3. Suares NC & Ford AC. Am J Gastroenterol. 2011 Sep;106(9):1582-91.

4. Mertz H, et al. Am J Gastroenterol. 1999;94(3):609-15.

5. Rao SS, et al. Am J Gastroenterol. 1998;93(7):1042-50.

6. Rao SSC, et al. J Clin Gastroenterol. 2004;38(8):680-5.

7. Nojkov B, et al. Am J Gastroenterol. 2019;114(11):1772-7.

8. Heaton KW, et al. Gut. 1992;33(6):818-24.

9. Prichard DO & Bharucha AE. 2018 Oct 15;7:F1000 Faculty Rev-1640.

10. Ashraf W, et al. Am J Gastroenterol. 1996;91(1):26-32.

11. Agachan F, et al.. Dis Colon Rectum. 1996;39(6):681-5.

12. Frank L, et al. Scand J Gastroenterol. 1999;34(9):870-7.

13. Marquis P, et al. Scand J Gastroenterol. 2005;40(5):540-51.

14. Yan Y, et al. Gastroenterology. 2020;158(6):S-400.

15. Rao SSC. Am J Gastroenterol. 2018;113(5):635-8.

16. Tantiphlachiva K, et al. Digital rectal examination is a useful tool for identifying patients with dyssynergia. Clin Gastroenterol Hepatol. 2010;8(11):955-60.

17. Carrington EV, et al. Nat Rev Gastroenterol Hepatol. 2018;15(5):309-23.

18. Tetangco EP, et al. Performing and analyzing high-resolution anorectal manometry. NeuroGastroLatam Rev. 2018;2:120-32.

19. Lee YY, et al. Curr Gastroenterol Rep. 2013;15(12):360.

20. Sharma M, et al. Neurogastroenterol Motil. 2020;32(10):e13910.

21. Rao SSC, et al.. Am J Gastroenterol. 2006;101(12):2790-6.

22. Carrington EV, et al. Neurogastroenterol Motil. 2020;32(1):e13679.

23. Rao SSC. Gastroenterol Clin North Am. 2008;37(3):569-86, viii.

24. Rao SSC, et al. Gastroenterology. 2016;150(4):S158-9.

25. Guinet A, et al. Int J Colorectal Dis. 2011;26(4):507-13.

26. Rattan S, et al. Gastroenterology. 1992;103(1):43-50.

27. Remes-Troche JM & Rao SSC. 2008;2(3):323-35.

28. Zaafouri H, et al..Int J Surgery. 2015. 2(1):9-17.

29. Remes-Troche JM, et al. Dis Colon Rectum. 2010;53(7):1047-54.

 

 

30. Rao SSC, et al. Gastroenterology. 2013;144(5):S-363.

31. Yu T, et al. Medicine (Baltimore). 2016;95(19):e3667.

32. Gladman MA, et al. Neurogastroenterol Motil. 2009;21(5):508-16, e4-5.

33. Lee KJ, et al. Digestion. 2006;73(2-3):133-41 .

34. Rao SSC, et al. Neurogastroenterol Motil. 2002;14(5):553-9.

35. Coss-Adame E, et al.. Clin Gastroenterol Hepatol. 2015;13(6):1143-1150.e1.

36. Chiarioni G, et al. Clin Gastroenterol Hepatol. 2014;12(12):2049-54.

37. Gu G, et al. Gastroenterology. 2018;154(6):S-545–S-546.

38. Savoye-Collet C, et al.. Gastroenterol Clin North Am. 2008;37(3):553-67, viii.

39. Videlock EJ, et al. Neurogastroenterol Motil. 2013;25(6):509-20.

40. Rao SSC, et al. Neurogastroenterol Motil. 2004;16(5):589-96.

41. Wald A, et al. Am J Gastroenterol. 2014;109(8):1141-57 ; (Quiz) 1058.

42. Rao SSC & Patcharatrakul T. J Neurogastroenterol Motil. 2016;22(3):423-35.

43. Rao SS, et al. Functional Anorectal Disorders. Gastroenterology. 2016. S0016-5085(16)00175-X.

44. Bharucha AE, et al.. Gastroenterology. 2013;144(1):218-38.

45. Voderholzer WA, et al. Am J Gastroenterol. 1997;92(1):95-8.

46. Lee HJ, et al. Neurogastroenterol Motil. 2015;27(6):787-95.

47. Simón MA & Bueno AM. J Clin Gastroenterol. 2017;51(10):e90-4.

48. Chiarioni G,et al.. Gastroenterology. 2006;130(3):657-64.

49. Heymen S, et al.. Dis Colon Rectum. 2007;50(4):428-41.

50. Rao SSC, et al. Clin Gastroenterol Hepatol. 2007;5(3):331-8.

51. Rao SSC, et al. Am J Gastroenterol. 2010;105(4):890-6.

52. Patcharatrakul T, et al. Biofeedback therapy. In Clinical and basic neurogastroenterology and motility. India: Stacy Masucci; 2020:517-32.

53. Huaman J-W, et al. Clin Gastroenterol Hepatol. 2020;18(11):2463-2470.e1.

54. Patcharatrakul T, et al. Clin Gastroenterol Hepatol. 2018;16(5):715-21.

55. Chaudhry A, et al. Gastroenterology. 2020;158(6):S-382–S-383.

56. Shim LSE, et al. Aliment Pharmacol Ther. 2011;33(11):1245-51.

57. Rao SSC, et al. Neurogastroenterol Motil. 2015;27(5):594-609.

58. Jangsirikul S, et al. Gastroenterology. 2020;158(6):S-383.

59. Rao SSC, et al. Am J Gastroenterol. 2019;114(6):938-44.

60. Ron Y, et al.. Dis Colon Rectum. 2001;44(12):1821-6.

61. Podzemny V, et al. World J Gastroenterol. 2015;21(4):1053-60.

62. Faried M, et al. J Gastrointest Surg. 2010;14(8):1235-43.

63. Hallan RI, et al. Lancet. 1988;2(8613):714-7.

Publications
Publications
Topics
Article Type
Sections
Disallow All Ads
Content Gating
No Gating (article Unlocked/Free)
Alternative CME
Disqus Comments
Default
Use ProPublica
Hide sidebar & use full width
render the right sidebar.
Conference Recap Checkbox
Not Conference Recap
Clinical Edge
Display the Slideshow in this Article
Medscape Article
Display survey writer
Reuters content
Disable Inline Native ads

Defecation Disorders: Diagnosis and Treatment

Article Type
Changed
Fri, 03/09/2018 - 11:25
Display Headline
Defecation Disorders: Diagnosis and Treatment

From the Digestive Health Center, Medical College of Georgia, Georgia Regents University, Augusta, GA

 

Defecation is a coordinated process that involves generation of sufficient propulsive forces in the abdomen and rectum together with relaxation of the puborectalis and external anal sphincter. Likewise, continence involves conscious retention of bowel contents until stool or gas can be voluntarily eliminated in an appropriate fashion. A failure of these processes leads to altered bowel function and disorders of defecation that are commonly encountered in clinical practice. They include a diverse group of maladies that result in altered defecation. Among them are functional disorders, such as dyssynergic defecation, and mechanical/structural disorders, such as rectocele, solitary rectal ulcer syndrome (SRUS), excessive perineal descent, and rectal prolapse. This article discusses 3 cases that illustrate the clinical features and management approaches to dyssynergic defecation, SRUS, and fecal incontinence.

Case Study 1

Presentation and History

A 26-year-old white woman with a 10-year history of constipation presents to a gastroenterologist after referral from her primary care physician. She reports spontaneous bowel movements once every 2 weeks, and often she has to induce stools by using enemas or suppositories. Stooling became progressively more difficult for her during her teenage years, with infrequent bowel movements and hard stools (type 1–2 on Bristol stool scale). She also reports having to strain excessively during bowel movements, and on average she spends 30 minutes in the bathroom. She denies experiencing any perianal pain or bleeding or using manual maneuvers to defecate, but she often feels a sense of incomplete evacuation. She also describes intermittent abdominal pain and bloating.

She has tried several over-the-counter laxatives, includ-ing milk of magnesia, senna, and magnesium citrate. Most recently, she tried lubiprostone and polyethylene glycol without improvement. Her past medical history is significant for endometriosis, exploratory laparotomy, and 1 vaginal delivery. There is no family history of colorectal cancer or inflammatory bowel disease. She works as a truck driver and does not use alcohol, illicit drugs, or tobacco. There is no history of physical or sexual abuse. Her current medications include lubiprostone 24 µg twice daily, polyethylene glycol 17 g twice daily, and a birth control pill.

Physical Examination

On physical examination, the patient appears healthy without any distress. Her body mass index is 26 kg/m2, and vital signs are normal. General examination is normal. Abdomen is flat, and bowel sounds are normal. Mild tenderness is noted in both lower quadrants. Rectal examination reveals normal anal skin folds. Digital exam-ination reveals a normal resting tone with pellet-like stool that is heme-negative. When asked to attempt defecation, she shows poor perineal descent and paradoxical contraction of the anal sphincter.

Laboratory Evaluation

Laboratory testing reveals normal levels of thyrotropin and thyroxine, no anemia on complete blood count, and normal levels of calcium, glucose, and electrolytes.

  • What are the possible causes for this patient’s altered bowel habits?

  • What is the approach to physical examination in patients with constipation?

Causes of Constipation

Constipation is a common digestive disorder, affecting up to 20% of the world’s population [1]. Primary or idiopathic constipation consists of 3 common overlapping subtypes: slow-transit constipation, dyssynergic defecation, and constipation-predominant irritable bowel syndrome. Slow-transit constipation involves the slow movement of stool through the colon. This is usually seen on a colonic transit study or with wireless motility capsule study. Dyssynergia in general is caused by functional outlet obstruction with or without normal colonic transit. Patients with dyssynergia often complain of incomplete evacuation, excessive straining, bloating, and blockage [2]. Often patients with dyssynergia resort to manual disimpaction/vaginal splinting and/or abdominal pressure to facilitate bowel movements. Secondary constipation may result from metabolic disorders (eg, hypercalcemia and hypokalemia, disorders associated with renal failure, hypothyroidism, and diabetes) as well as medications, including narcotics, anticholinergics, and antidepressants.

Rectal Examination

Physical examination in patients with constipation should include a detailed rectal examination. The perianal skin should be inspected closely for fissures, fistulae, and skin excoriation. The anocutaneous reflex should be checked along with resting and squeeze anal tone. A study by Rao et al[3] showed that rectal examination could identify 76% of patients with dyssynergia. The sensitivity and positive predictive value for diagnosing dyssynergia with digital rectal examination was 81% and 99%, respectively, making it a good screening test for dyssynergia [3].

  • When is colonoscopy indicated in the workup of constipation?

  • What imaging studies may be useful?

Colonoscopy

Colonoscopic evaluation is only indicated in patients with alarming features such as rectal bleeding, weight loss, unex-plained abdominal pain, palpable mass in the abdomen or rectum, persistent and unexplained anal/rectal pain, or anemia, as well as in patients over age 50 years [4].

Colonic Transit Study

Two imaging studies can be useful in the evaluation of a patient with constipation: colonic transit study and defeco-graphy. A colonic transit study provides useful information regarding the rate at which stool travels through the colon. This test is performed by administering one capsule (Sitzmarks, Konsyl Pharmaceuticals, Fort Worth, TX) containing radiopaque markers. A plain radiograph of the abdomen is obtained on day 6 (120 hr after ingestion of capsule). A transit study is considered abnormal if more than 20% of markers (> 5) are present on a plain radiograph of the abdomen. Approximately two-thirds of patients with dyssynergia have an abnormal colonic transit study, with retention of markers either in the rectosigmoid region or throughout the colon [5]. Wireless motility capsule is a newer test that is comprised of ingesting a capsule and wearing a recorder for up to 5 days. This test measures regional transit (ie, gastric emptying, colonic transit time, and whole gut transit time), is standardized and validated, and avoids use of radiation [6].

Defecography

Defecography is conducted by instilling a barium paste in the rectum and monitoring evacuation of the barium radiologically. It can reveal poor activation of the levator ani muscles, prolonged retention of the barium, inability to expel the barium, absence of a striping wave, rectal mucosal intussusception, rectocele, abnormal perineal descent, or rectal prolapse [5]. Although abnormalities are frequently found on defecography, they may not translate into clinical dysfunction. In one study, 77% of women with complaints of defecation disorders had abnormalities on defecography, but there was no relationship between the abnormalities and the patients’ symptoms [7]. Hence, defecography is not recommended unless there is clinical suspicion of prolapse or excessive descent. Endoanal and dynamic pelvic magnetic resonance imaging (MRI) can evaluate global pelvic floor anatomy in dynamic function [8]. Dynamic MRI in the seated position provides the most physiologic approach.

  • What testing is needed to make a diagnosis of dyssynergic defecation?

Both an abnormal balloon expulsion test and an abnormal pattern of defecation on anal rectal manometry are required to diagnose dyssynergic defecation [9]. Anorectal manometry provides information regarding rectal and anal pressures at rest and during maneuvers of simulated defecation as well as information on rectal sensation, rectoanal reflexes, and compliance [2,10]. There are 4 patterns of dyssynergia found on anorectal manometry: type 1, normal push effort with paradoxical contraction of the anal sphincter; type 2, poor push effort with paradoxical contraction of the anal sphincter; type 3, normal push effort with incomplete or absent relaxation of the anal sphincter; and type 4, poor push with incomplete anal relaxation. The balloon expulsion test should be included in the work-up of dyssynergia.

Normal subjects can expel a 50-mL water-filled balloon in less than 1 minute. Although normal patients can show a dyssynergic pattern in the left lateral decubitus position, when seated on a commode and with a sensation of stooling most exhibit a normal pattern of defecation [9].

Diagnosis

      The case patient undergoes a colonic transit study and has several retained markers distributed primarily in the left colon (Figure 1). Defecography shows incomplete relaxation of the puborectalis, with only 10% of the rectal contents expelled during the study. The results of anorectal manometry and the balloon expulsion test reveal normal sphincter pressures but evidence for an abnormal defecation pattern or dyssynergia: low defecation index (calculated by dividing the rectal pressure by the anal residual pressure during straining) and prolonged balloon expulsion time (Table 1). Based on these findings, a diagnosis of dyssynergic defecation is made.

 

  • What treatment options are available for dyssynergia?

 

The treatment of patients with dyssynergic defecation consists of standard therapies for constipation, including diet, laxatives, and timed toileting. Medical therapy includes laxatives, polyethylene glycol, and lubiprostone. 

Biofeedback therapy is useful and efficacious. Its purpose is to restore a normal pattern of defecation by using an instrument-based education program. The primary goals are to correct dyssynergia and improve rectal sensory perception. Diaphragmatic muscle training, simulated defecation, and manometry- or electromyography-guided anal sphincter and pelvic floor relaxation techniques will improve symptoms in more than 70% of patients [11–13]. Table 2 summarizes data from recent published trials on the efficacy of biofeedback [11–14]. Other avenues of treatment include botulinum toxin injection and surgical approaches that are generally ineffective [15,16].

Case Study 2

Initial Presentation and History

A 39-year-old woman presents with a 5-year history of intermittent bright red blood with stooling. Most often, she notices blood on the toilet paper or when wiping and rarely in the commode. She reports having experienced difficulty with bowel movements since her teens. She does not have a daily urge but strains up to 30 minutes to pass stool that is hard in consistency (type 1–2 on the Bristol stool scale). Over the past year, she has started using fingers to remove stool.

The patient reports bloating and abdominal discomfort that is improved with stooling. Her weight has been stable. Current medications include polyethylene glycol 17 g twice daily, sodium docusate 100 mg twice daily, iron sulfate 325 mg 3 times daily, and a birth control pill. Her past medical history is significant for iron deficiency anemia. Family history is notable for her mother and sister with similar “bowel troubles,” but no family history of inflammatory bowel disease or colorectal cancer. She is a salesperson and has been married for 7 years. She does not use tobacco or alcohol. As a child, she was sexually abused. She did not receive any formal counseling for the abuse. Review of systems is negative.

Physical Examination

General and neurologic examinations are normal. The abdomen is mildly distended, bowel sounds are normal, there is mild tenderness, and stool is palpable in the left lower quadrant. Rectal examination reveals normal anal skin with no fissures, intact anocutaneous reflex, and hard stool in the rectal vault that is guaiac-positive. The resting anal sphincter tone is elevated, and when asked to attempt defecation, there is excessive perineal descent and rectal mucosal intussusception with paradoxical anal contraction.

Laboratory Evaluation and Endoscopy

Laboratory testing reveals normal hemoglobin, white blood cell count, mean corpuscular volume, and electrolytes. Endoscopy is performed and reveals an irregular solitary ulcer base covered in fibrinous exudates in the rectum (Figure 2).

  •  What is SRUS and how is it diagnosed?

Evaluation and Diagnosis

SRUS is characterized by single or multiple ulcerations of the rectal mucosa along with distinct pathologic changes [17]. The term solitary rectal ulcer is a misnomer because many patients have more than 1 lesion, and it is not always an ulcer. Patients with SRUS present with several symptoms, but the most common is passage of blood or mucus, and up to 26% may be asymptomatic [18]. The pathophysiology of this condition is poorly understood. Multiple mechanisms have been implicated, including occult or overt rectal prolapse, dyssynergia, rectal mucosal intussusception, rectal hypersensitivity with a persistent feeling of a need to defecate, and reduced mucosal blood flow [19].

The diagnosis of SRUS is based on the patient’s clinical history combined with endoscopy and histopathology findings. Endoscopically, the lesions may vary in appearance. Shallow ulcerations on hyperemic surrounding mucosa located on the anterior wall is the most common finding [17]. Lesions vary in size, although most are 1 to 1.5 cm in diameter [17] and rarely involve more than half the circumference of the rectal wall. Polypoid lesions occur in approximately 25% of patients with SRUS, and multiple lesions occur in 30% [17].

Obtaining specimens for histology is an important step in the evaluation of SRUS. The differential diagnosis includes Crohn’s disease, ulcerative colitis, ischemic colitis, and malignancy. The typical histologic findings include fibromuscular hyperplasia with smooth muscle infiltration of the lamina propria, thickening of the muscularis mucosa, regenerative changes, and distortion of the crypt architecture [17].

  • Are physiologic or imaging studies helpful in the diagnosis of SRUS?

Two complementary physiologic tests for SRUS are anorectal manometry and defecography. Anorectal manometry often shows evidence of dyssynergia and rectal hypersensitivity in patients with SRUS [20,21]. Hyper-sensitivity may produce a sensation of incomplete evacuation, which in turn results in excessive straining. Defecography may reveal rectal mucosal intussusception or overt rectal prolapse. The patient in this case had evidence of rectal hypersensitivity on anorectal manometry along with excessive perineal descent on defecography.

  • What are treatment options for SRUS?

Treatment of SRUS is not standardized. The options include topical medical therapy, biofeedback, and surgery. Uncontrolled studies have suggested that 5-aminosalicylic acid enema [22], sucralfate enema [23], steroid enema [24], and fibrin glue [25] may improve symptoms. Patients who fail topical therapy and have evidence of dyssynergia on anorectal manometry should receive biofeedback therapy. A case-control study of biofeedback involving 11 patients with refractory SRUS and 15 healthy controls showed improvement in anorectal function, including dyssynergia [21]. At follow-up endoscopy, 36% had complete mucosal healing and more than 50% showed partial healing. In a study involving 16 patients with SRUS and 26 healthy controls, Jarrett et al [26] showed that 75% of patients who underwent biofeedback therapy had improved and 31% had ulcer resolution. Surgical therapy should be considered in rare patients who are refractory to medical therapy. The Delorme procedure is commonly performed with a success rate of 42% to 100% [27].

The case patient underwent biofeedback therapy, and after 5 sessions had complete healing of the lesion and resolution of rectal bleeding and bowel symptoms.

Case Study 3

Initial Presentation and History

A 75-year-old woman is referred to a gastroenterologist with complaints of incomplete stool evacuation and intermittent fecal seepage. She passes stools daily but sits on the toilet for 15 to 20 minutes, and after straining will pass only a small amount of stool. She describes stools as type 4 on the Bristol scale with no blood or mucus. One to 2 hours after a bowel movement, she experiences some wetness in the perineal region and upon checking often notices that a tablespoon full of stool material has leaked out. Sometimes, she will pass another large stool. She denies any leakage of stool while sleeping. Occasionally, she has urgency and leaks stool before reaching the toilet. In the past, she has used digital maneuvers to facilitate stooling. This problem has interfered with shopping, socializing, and taking vacations.

Her past medical history is significant for narcolepsy, hypertension, tubal ligation, appendectomy, and inguinal hernia repair. Obstetric history is significant for 6 vaginal deliveries, 1 requiring episiotomy but no forceps use. Her current medications include estradiol vaginal cream, hydrochlorothiazide, pilocarpine, and amitriptyline 10 mg 3 times daily. She also reports stress urinary incontinence, particularly with sneezing and coughing.

Physical Examination

Physical examination reveals a well-nourished woman with normal vital signs and a normal general examination. Abdominal examination is normal. A rectal examination shows no fissures, but the anocutaneous reflex is absent on the right side. Resting and squeeze sphincter tones are normal, with good perineal descent and normal anal relaxation.

Laboratory Evaluation

Thyroid function tests, complete blood count, and liver biochemistry tests are normal. Results of anorectal manometry are shown in Table 3. Defecography shows a slightly larger resting anorectal angle and an anterior rectocele (Figure 3). Anal endosonography reveals a large sphincter defect in the external anal sphincter (Figure 4).

 

  • What are the mechanisms involved in fecal incontinence?

  • What are the 3 clinical subtypes of fecal incontinence?

Mechanisms and Subtypes

Fecal incontinence is often an unvoiced problem that causes significant social stigma. Approximately 2% of the US population suffers from fecal incontinence [28], with a higher prevalence among women and elderly persons. Several mechanisms are involved in the pathogenesis of fecal incontinence. A common cause is injury to the external or internal anal sphincter, puborectalis muscle, or pudendal nerves, often after obstetric trauma. Hence, a detailed obstetric history including number of vaginal deliveries, use of forceps, tears, and episiotomy is important. Sphincter disruption, most commonly after surgery for hemorrhoid or anal fissure, can result in incontinence. Likewise, reduced rectal compliance causes urgency and fecal incontinence. Impaired rectal sensation results in the accumulation of stool and overflow. Patients rarely have a single cause, with 80% having more than one factor that leads to incontinence [29].

Clinically, fecal incontinence can be classified into 3 categories. Urge incontinence is characterized by the inability to control stool discharge despite active attempts to retain contents. These patients often have disruption or injury to the external anal sphincter. Fecal seepage is the involuntary discharge of less than 2 tablespoons of stool matter without awareness. Seepage can result from impaired rectal evacuation and dyssynergia. Often patients with seepage complain of incomplete evacuation. Passive incontinence refers to the involuntary discharge of stool contents without awareness. These patients often have underlying neuropathy and sphincter weakness [30,31].

  • What is the approach to evaluation and diagnosis?

Evaluation and Diagnosis

Physical examination of patients with fecal incontinence should include a detailed rectal examination, similar to the exam performed in patients who present with constipation. It should include perineal inspection for fissures, fistulae, and skin excoriation. The anocutaneous reflex should be checked along with the resting and squeeze sphincter tone and sphincter relaxation. Further investigations should focus on determining the underlying mechanism in order to facilitate treatment.

Endoscopic investigation should be performed to exclude mucosal disease or malignancy. Anorectal manometry provides objective information regarding resting and squeeze anal sphincter tone, rectal compliance, rectal sensitivity, and rectoanal reflexes [29]. Some experts believe that anorectal manometry is not needed for diagnosis and emphasize the importance of rectal examination and history [32]. Proponents of anorectal manometry point out the importance of physiologic data that can be gained and how it may direct therapy. For example, anorectal manometry and sensory testing may reveal weak anal sphincters and impaired rectal sensation. The latter cannot be identified by clinical evaluation alone. These 2 pathophysiologic findings could enable the biofeedback therapist to focus on improving both anal sphincter tone and rectal sensation [33]. Defecography may reveal anterior rectocele, mucosal intussusception, or rectal prolapse. Anal ultrasound provides information on the structural integrity of the external and internal anal sphincters [34]. Ultrasound is widely available and is relatively inexpensive. Endoanal MRI may provide better information regarding the integrity of the external anal sphincter [35].

  • What are the treatment options?

The goal of treatment is to restore continence and quality of life. General considerations include stool bulking agents such as fiber supplements. Antidiarrheal agents, such as loperamide and diphenoxylate/atropine, are useful as they can decrease stool volume and increase and prolong sphincter pressure and colonic transit time [36,37]. Patients with diarrhea and functional incontinence may benefit from treatment with cholestyramine [38]. Biofeedback therapy improves sphincter tone and rectal sensation [39]. The number of biofeedback sessions is titrated to the patient’s needs, but often 6 sessions are required [40]. Generally, a 70% success rate has been described. Table 4 summarizes recent evidence supporting the use of biofeedback in the treatment of fecal incontinence [41–46].

Surgery for incontinence should be reserved for patients who have failed aggressive conservative management and biofeedback therapy. Overlapping sphincteroplasty is the most common surgery performed for fecal incontinence, with a success rate between 35% and 70% [47,48]. Creation of a neosphincter via dynamic graciloplasty or artificial sphincter has been tried in patients with an irreversibly damaged anal sphincter, but the success rate is low and the complication rate is high [49].

Sacral nerve stimulation (SNS) involves inserting electrodes in the lower back and connecting them to a pulse generator that produces pulses of electricity that innervate the nerves controlling the anal sphincters. Two double-blind crossover studies have reported a beneficial effect of SNS in fecal incontinence [50,51]. In 19 patients who preferred the periods when the stimulator was turned on, the median number of fecal incontinence episodes per week decreased from 1.7 to 0.7, and in the 5 patients who preferred the off period, the median number of fecal incontinence episodes per week increased from 1.7 to 3.7. SNS is now approved by FDA and insurance payers. Recently, hyaluronic acid/dextranomer injection (Solesta, Salix Pharmaceuticals, Raleigh, NC) has also been approved by FDA and has been shown to improve incontinence. A randomized controlled trial showed a 52% response rate to hyaluronic acid/dextranomer compared to a 31% response with placebo [52].

Conclusion

The 3 cases presented illustrate the complexities of several common anorectal disorders. A definitive diagnosis can be established in patients with defecation disorders through systematic evaluations and physiologic and imaging studies. Diagnosis in turn can pave the way for appropriate medical, behavioral, or surgical treatment. If facilities for appropriate testing are unavailable, it is important to refer these patients to appropriate specialists instead of embarking on empirical therapies which may prove futile. Treatment is often possible, and in a majority of patients their symptoms can be ameliorated.

Corresponding author: Satish S.C. Rao, MD, PhD, Section of Gastroenterology and Hepatology, Medical College of Georgia, Georgia Regents University, BB R2540, 1120 15th St., Augusta, GA 30912.

Funding/support: Portions of this work were supported by National Institutes of Health grant RO1 DK 57100-05.

References

1. Higgins PD, Johanson JF. Epidemiology of constipation in North America: a systematic review. Am J Gastroenterol 2004;99:750–9.

2. Rao SS, Mudipalli RS, Stessman M, Zimmerman B. Investigation of the utility of colorectal function tests and Rome II criteria in dyssynergic defecation (Anismus). Neurogastroenterol Motil 2004;16:580–96.

3. Rao SS, Tantiphlachiva K, Rao PS, et al. How useful is digital rectal examination in the assessment of patients with dyssynergia? Am J Gastroenterol 2007;S268:419.

4. Pepin C, Ladabuam U. The yield of lower endoscopy in patients with constipation: survey of a university hospital, public county hospital, and a Veterans Administration medical center. Gastrointest Endosc 2002;56:325–32.

5. Rao SS. Constipation: evaluation and treatment. Gastroenterol Clin North Am 2003;32:659–83.

6. Rao SS, Camilleri M, Hasler WL, et al. Evaluation of  gastrointestinal transit in clinical practice: position paper of the  American and European Neurogastroenterology and Motility Societies. Neurogastroenterol Motil; 2011; 23: 8-23.

7. Savoye-Collet C, Savoye G, Koning E, et al. Defecography in symptomatic older women living at home. Age Ageing 2003;32:347–50.

8. Bolog N, Weishaupt D. Dynamic MR imaging of outlet obstruction. Rom J Gastroenterol 2005;14:293–302.

9. Rao SS, Kavlock R, Rao S. Influence of body position and stool characteristics on defecation in humans. Am J Gastroenterol 2006;101:2790–6.

10. Karlbom U, Lundin E, Graf W, Pahlman L. Anorectal physiology in relation to clinical subgroups of patients with severe constipation. Colorectal Dis 2004;6:343–9.

11. Chiarioni G, Whitehead WE, Pezza V, et al. Biofeedback is superior to laxatives for normal transit constipation due to pelvic floor dyssynergia. Gastroenterology 2006;130:657–64.

12. Rao SS, Seaton K, Miller M, et al. Randomized controlled trial of biofeedback, sham feedback, and standard therapy for dyssynergic defecation. Clin Gastroenterol Hepatol 2007;5:331–8.

13. Chiarioni G, Saladini L, Whitehead W. Biofeedback benefits only patients with outlet dysfunction, not patients with isolated slow transit constipation. Gastroenterology 2005;129:86–97.

14. Heymen S, Scarlett Y, Jones K, et al. Randomized controlled trial shows biofeedback to be superior to alternate treatments for patients with pelvic floor dyssynergia-type defecation [abstract]. Am J Gastroenterol 2005;100:S335.

15. Hallan RI, Williams NS, Melling J, et al. Treatment of anismus in intractable constipation with botulinum A toxin. Lancet 1988;2:714–7.

16. Boccasanta P, Venturi M, Stuto A, et al. Stapled transanal rectal resection for outlet obstruction: a prospective, multicenter trial. Dis Colon Rectum 2004;47:1285–97.

17. Sharara AI, Azar C, Amr SS, et al. Solitary rectal ulcer syndrome: endoscopic spectrum and review of the literature. Gastrointest Endosc 2005;62:755–62.

18. Tjandra JJ, Fazio VW, Church JM, et al. Clinical conundrum of solitary rectal ulcer. Dis Colon Rectum 1992;35:227–34.

19. Felt-Bersma RJ, Cuesta A. Rectal prolapse, rectal intussusception, rectocele, and solitary rectal ulcer syndrome. Gastroenterol Clin North Am 2001;30:199–222.

20. Vaizey CJ, van den Bogaerde JB, Emmanuel AV, et al. Solitary rectal ulcer syndrome. Br J Surg 1998;85:1617–23.

21. Rao SS, Ozturk R, De Ocampo S, Stessman M. Pathophysiology and role of biofeedback therapy in solitary rectal ulcer syndrome. Am J Gastroenterol 2006;101:613–8.

22. Kumar M, Puri AS, Srivastava R, Yachha SK. Solitary rectal ulcer syndrome in a child treated with local sulfasalazine. Indian Pediatr 1994;31:1553–5.

23. Zargar SA, Khuroo MS, Mahajan R. Sucralfate retention enemas in solitary rectal ulcer. Dis Colon Rectum 1991;34:455–7.

24. Bishop PR, Nowicki MJ. Nonsurgical therapy for solitary rectal ulcer syndrome. Curr Treat Options Gastroenterol 2002;5:215–23.

25. Ederle A, Bulighin G, Orlandin PG, Pilati S. Endoscopic application of human fibrin sealant in the treatment of solitary rectal ulcer syndrome [letter]. Endoscopy 1992;24:736–7.

26. Jarrett ME, Emmanuel AV, Vaizey CJ, Kamm MA. Behavioural therapy (biofeedback) for solitary rectal ulcer syndrome improves symptoms and mucosal blood flow. Gut 2004;53:368–70.

27. Tweedie DJ, Varma JS. Long-term outcome of laparoscopic mesh rectopexy for solitary rectal ulcer syndrome. Colorectal Dis 2005;7:151–5.

28. Rao SS. Diagnosis and management of fecal incontinence. American College of Gastroenterology Practice Parameters Committee. Am J Gastrenterol 2004;99:1585–604.

29. Rao SS, Patel RS. How useful are manometric tests of anorectal function in the management of defecation disorders? Am J Gastroenterol 1997;92:469–75.

30. Rao SS. Pathophysiology of adult fecal incontinence. Gastroenterology 2004;126(1 Suppl 1):S14–22.

31. Deutekom M, Dobben AC, Terra MP, et al. Clinical presentation of fecal incontinence and anorectal function: what is the relationship? Am J Gastroenterol 2007;102:351–61.

32. Wald A. Con: anorectal manometry and imaging are not necessary in patients with fecal incontinence. Am J Gastroenterol 2006;101:2681–3.

33. Bharucha AE. Pro: anorectal testing is useful in fecal incontinence. Am J Gastroenterol 2006;101:2679–81.

34. Tuteja AK, Rao SS. Review article: recent trends in diagnosis and treatment of faecal incontinence. Aliment Pharmacol Ther 2004;19:829–40.

35. Terra MP, Beets-Tan RG, van der Hulst VP, et al. MRI in evaluating atrophy of the external anal sphincter in patients with fecal incontinence. AJR Am J Roentgenol 2006;187:991–9.

36. Sun WM, Read NW, Verlinden M. Effects of loperamide oxide on gastrointestinal transit time and anorectal function in patients with chronic diarrhoea and faecal incontinence. Scan J Gastroenterol 1997;32:34–8.

37. Harford WV, Krejs GJ, Santa Ana CA, Fordtran JS. Acute effect of diphenoxylate with atropine (Lomotil) in patients with chronic diarrhea and fecal incontinence. Gastroenterol 1980;78:440–3.

38. Remes-Troche JM, Ozturk R, Philips C, et al. Cholesteryramine—a useful adjunct for the treatment of patients with fecal incontinence. Int J Colorectal Dis 2008;23:189–99.

39. Rao SS. The technical aspects of biofeedback therapy for defecation disorders. Gastroenterologist 1998;6:96–103.

40. Rao SS, Welcher KD, Happel J. Can biofeedback therapy improve anorectal function in fecal incontinence? Am J Gastroenterol 1966;91:2360–6.

41. Byrne CM, Solomon MJ, Young JM, et al. Biofeedback for fecal incontinence: short-term outcomes of 513 consecutive patients and predictors of successful treatment. Dis Colon Rectum 2007;50:417–27.

42. Norton C, Chelvanayagam S, Wilson-Barnett J, et al. Randomized controlled trial of biofeedback for fecal incontinence. Gastroenteroloy 2003;125:1320–9.

43. Heymen S, Whitehead W. EMG biofeedback vs. Kegel exercise for the treatment of fecal incontinence. Gastroenterology 2007;132:A–83.

44. Pager CK, Solomon MJ, Rex J, Roberts RA. Long-term outcomes of pelvic floor exercise and biofeedback treatment for patients with fecal incontinence. Dis Colon Rectum 2002;45:997–1003.

45. Ozturk R, Niazi S, Stessman M, Rao SS. Long-term and objective changes of anorectal function after biofeedback therapy for faecal incontinence. Aliment Pharmacol Ther 2004;20:667–74.

46. Byrne CM, Solomon MJ, Rex J, et al. Telephone vs. face-to-face biofeedback for fecal incontinence: comparison of two techniques in 239 patients. Dis Colon Rectum 2005;48:2281–8.

47. Zorcolo L, Covotta L, Bartolo DC. Outcome of anterior sphincter repair for obstetric injury: comparison of early and late results. Dis Colon Rectum 2005;48:524–31.

48. Madoff RD. Surgical treatment options for fecal incontinence. Gastroenterology 2004;126(1 Suppl 1)S48–54.

49. Thornton MJ, Kennedy ML, Lubowski DZ, King DW. Long-term follow-up of dynamic graciloplasty for faecal incontinence. Colorectal Dis 2004;6:470–6.

50. Leroi AM, Parc Y, Lehur PA, et al. Efficacy of sacral nerve stimulation for fecal incontinence: results of a multicenter double-blind crossover study. Ann Surg 2005;242:662–9.

51. Vaizey CJ, Kamm MA, Nicholls RJ. Double-blind crossover study of sacral nerve stimulation for fecal incontinence. Dis Colon Rectum 2000;43:298–302.

52. Graf W, Mellgren A, Matzel KE, et al. . Efficacy of dextranomer in stabilised hyaluronic acid for treatment of faecal incontinence: a randomised, sham-controlled trial. Lancet;2011;377:997–1003.

Issue
Journal of Clinical Outcomes Management - OCTOBER 2014, VOL. 21, NO. 10
Publications
Topics
Sections

From the Digestive Health Center, Medical College of Georgia, Georgia Regents University, Augusta, GA

 

Defecation is a coordinated process that involves generation of sufficient propulsive forces in the abdomen and rectum together with relaxation of the puborectalis and external anal sphincter. Likewise, continence involves conscious retention of bowel contents until stool or gas can be voluntarily eliminated in an appropriate fashion. A failure of these processes leads to altered bowel function and disorders of defecation that are commonly encountered in clinical practice. They include a diverse group of maladies that result in altered defecation. Among them are functional disorders, such as dyssynergic defecation, and mechanical/structural disorders, such as rectocele, solitary rectal ulcer syndrome (SRUS), excessive perineal descent, and rectal prolapse. This article discusses 3 cases that illustrate the clinical features and management approaches to dyssynergic defecation, SRUS, and fecal incontinence.

Case Study 1

Presentation and History

A 26-year-old white woman with a 10-year history of constipation presents to a gastroenterologist after referral from her primary care physician. She reports spontaneous bowel movements once every 2 weeks, and often she has to induce stools by using enemas or suppositories. Stooling became progressively more difficult for her during her teenage years, with infrequent bowel movements and hard stools (type 1–2 on Bristol stool scale). She also reports having to strain excessively during bowel movements, and on average she spends 30 minutes in the bathroom. She denies experiencing any perianal pain or bleeding or using manual maneuvers to defecate, but she often feels a sense of incomplete evacuation. She also describes intermittent abdominal pain and bloating.

She has tried several over-the-counter laxatives, includ-ing milk of magnesia, senna, and magnesium citrate. Most recently, she tried lubiprostone and polyethylene glycol without improvement. Her past medical history is significant for endometriosis, exploratory laparotomy, and 1 vaginal delivery. There is no family history of colorectal cancer or inflammatory bowel disease. She works as a truck driver and does not use alcohol, illicit drugs, or tobacco. There is no history of physical or sexual abuse. Her current medications include lubiprostone 24 µg twice daily, polyethylene glycol 17 g twice daily, and a birth control pill.

Physical Examination

On physical examination, the patient appears healthy without any distress. Her body mass index is 26 kg/m2, and vital signs are normal. General examination is normal. Abdomen is flat, and bowel sounds are normal. Mild tenderness is noted in both lower quadrants. Rectal examination reveals normal anal skin folds. Digital exam-ination reveals a normal resting tone with pellet-like stool that is heme-negative. When asked to attempt defecation, she shows poor perineal descent and paradoxical contraction of the anal sphincter.

Laboratory Evaluation

Laboratory testing reveals normal levels of thyrotropin and thyroxine, no anemia on complete blood count, and normal levels of calcium, glucose, and electrolytes.

  • What are the possible causes for this patient’s altered bowel habits?

  • What is the approach to physical examination in patients with constipation?

Causes of Constipation

Constipation is a common digestive disorder, affecting up to 20% of the world’s population [1]. Primary or idiopathic constipation consists of 3 common overlapping subtypes: slow-transit constipation, dyssynergic defecation, and constipation-predominant irritable bowel syndrome. Slow-transit constipation involves the slow movement of stool through the colon. This is usually seen on a colonic transit study or with wireless motility capsule study. Dyssynergia in general is caused by functional outlet obstruction with or without normal colonic transit. Patients with dyssynergia often complain of incomplete evacuation, excessive straining, bloating, and blockage [2]. Often patients with dyssynergia resort to manual disimpaction/vaginal splinting and/or abdominal pressure to facilitate bowel movements. Secondary constipation may result from metabolic disorders (eg, hypercalcemia and hypokalemia, disorders associated with renal failure, hypothyroidism, and diabetes) as well as medications, including narcotics, anticholinergics, and antidepressants.

Rectal Examination

Physical examination in patients with constipation should include a detailed rectal examination. The perianal skin should be inspected closely for fissures, fistulae, and skin excoriation. The anocutaneous reflex should be checked along with resting and squeeze anal tone. A study by Rao et al[3] showed that rectal examination could identify 76% of patients with dyssynergia. The sensitivity and positive predictive value for diagnosing dyssynergia with digital rectal examination was 81% and 99%, respectively, making it a good screening test for dyssynergia [3].

  • When is colonoscopy indicated in the workup of constipation?

  • What imaging studies may be useful?

Colonoscopy

Colonoscopic evaluation is only indicated in patients with alarming features such as rectal bleeding, weight loss, unex-plained abdominal pain, palpable mass in the abdomen or rectum, persistent and unexplained anal/rectal pain, or anemia, as well as in patients over age 50 years [4].

Colonic Transit Study

Two imaging studies can be useful in the evaluation of a patient with constipation: colonic transit study and defeco-graphy. A colonic transit study provides useful information regarding the rate at which stool travels through the colon. This test is performed by administering one capsule (Sitzmarks, Konsyl Pharmaceuticals, Fort Worth, TX) containing radiopaque markers. A plain radiograph of the abdomen is obtained on day 6 (120 hr after ingestion of capsule). A transit study is considered abnormal if more than 20% of markers (> 5) are present on a plain radiograph of the abdomen. Approximately two-thirds of patients with dyssynergia have an abnormal colonic transit study, with retention of markers either in the rectosigmoid region or throughout the colon [5]. Wireless motility capsule is a newer test that is comprised of ingesting a capsule and wearing a recorder for up to 5 days. This test measures regional transit (ie, gastric emptying, colonic transit time, and whole gut transit time), is standardized and validated, and avoids use of radiation [6].

Defecography

Defecography is conducted by instilling a barium paste in the rectum and monitoring evacuation of the barium radiologically. It can reveal poor activation of the levator ani muscles, prolonged retention of the barium, inability to expel the barium, absence of a striping wave, rectal mucosal intussusception, rectocele, abnormal perineal descent, or rectal prolapse [5]. Although abnormalities are frequently found on defecography, they may not translate into clinical dysfunction. In one study, 77% of women with complaints of defecation disorders had abnormalities on defecography, but there was no relationship between the abnormalities and the patients’ symptoms [7]. Hence, defecography is not recommended unless there is clinical suspicion of prolapse or excessive descent. Endoanal and dynamic pelvic magnetic resonance imaging (MRI) can evaluate global pelvic floor anatomy in dynamic function [8]. Dynamic MRI in the seated position provides the most physiologic approach.

  • What testing is needed to make a diagnosis of dyssynergic defecation?

Both an abnormal balloon expulsion test and an abnormal pattern of defecation on anal rectal manometry are required to diagnose dyssynergic defecation [9]. Anorectal manometry provides information regarding rectal and anal pressures at rest and during maneuvers of simulated defecation as well as information on rectal sensation, rectoanal reflexes, and compliance [2,10]. There are 4 patterns of dyssynergia found on anorectal manometry: type 1, normal push effort with paradoxical contraction of the anal sphincter; type 2, poor push effort with paradoxical contraction of the anal sphincter; type 3, normal push effort with incomplete or absent relaxation of the anal sphincter; and type 4, poor push with incomplete anal relaxation. The balloon expulsion test should be included in the work-up of dyssynergia.

Normal subjects can expel a 50-mL water-filled balloon in less than 1 minute. Although normal patients can show a dyssynergic pattern in the left lateral decubitus position, when seated on a commode and with a sensation of stooling most exhibit a normal pattern of defecation [9].

Diagnosis

      The case patient undergoes a colonic transit study and has several retained markers distributed primarily in the left colon (Figure 1). Defecography shows incomplete relaxation of the puborectalis, with only 10% of the rectal contents expelled during the study. The results of anorectal manometry and the balloon expulsion test reveal normal sphincter pressures but evidence for an abnormal defecation pattern or dyssynergia: low defecation index (calculated by dividing the rectal pressure by the anal residual pressure during straining) and prolonged balloon expulsion time (Table 1). Based on these findings, a diagnosis of dyssynergic defecation is made.

 

  • What treatment options are available for dyssynergia?

 

The treatment of patients with dyssynergic defecation consists of standard therapies for constipation, including diet, laxatives, and timed toileting. Medical therapy includes laxatives, polyethylene glycol, and lubiprostone. 

Biofeedback therapy is useful and efficacious. Its purpose is to restore a normal pattern of defecation by using an instrument-based education program. The primary goals are to correct dyssynergia and improve rectal sensory perception. Diaphragmatic muscle training, simulated defecation, and manometry- or electromyography-guided anal sphincter and pelvic floor relaxation techniques will improve symptoms in more than 70% of patients [11–13]. Table 2 summarizes data from recent published trials on the efficacy of biofeedback [11–14]. Other avenues of treatment include botulinum toxin injection and surgical approaches that are generally ineffective [15,16].

Case Study 2

Initial Presentation and History

A 39-year-old woman presents with a 5-year history of intermittent bright red blood with stooling. Most often, she notices blood on the toilet paper or when wiping and rarely in the commode. She reports having experienced difficulty with bowel movements since her teens. She does not have a daily urge but strains up to 30 minutes to pass stool that is hard in consistency (type 1–2 on the Bristol stool scale). Over the past year, she has started using fingers to remove stool.

The patient reports bloating and abdominal discomfort that is improved with stooling. Her weight has been stable. Current medications include polyethylene glycol 17 g twice daily, sodium docusate 100 mg twice daily, iron sulfate 325 mg 3 times daily, and a birth control pill. Her past medical history is significant for iron deficiency anemia. Family history is notable for her mother and sister with similar “bowel troubles,” but no family history of inflammatory bowel disease or colorectal cancer. She is a salesperson and has been married for 7 years. She does not use tobacco or alcohol. As a child, she was sexually abused. She did not receive any formal counseling for the abuse. Review of systems is negative.

Physical Examination

General and neurologic examinations are normal. The abdomen is mildly distended, bowel sounds are normal, there is mild tenderness, and stool is palpable in the left lower quadrant. Rectal examination reveals normal anal skin with no fissures, intact anocutaneous reflex, and hard stool in the rectal vault that is guaiac-positive. The resting anal sphincter tone is elevated, and when asked to attempt defecation, there is excessive perineal descent and rectal mucosal intussusception with paradoxical anal contraction.

Laboratory Evaluation and Endoscopy

Laboratory testing reveals normal hemoglobin, white blood cell count, mean corpuscular volume, and electrolytes. Endoscopy is performed and reveals an irregular solitary ulcer base covered in fibrinous exudates in the rectum (Figure 2).

  •  What is SRUS and how is it diagnosed?

Evaluation and Diagnosis

SRUS is characterized by single or multiple ulcerations of the rectal mucosa along with distinct pathologic changes [17]. The term solitary rectal ulcer is a misnomer because many patients have more than 1 lesion, and it is not always an ulcer. Patients with SRUS present with several symptoms, but the most common is passage of blood or mucus, and up to 26% may be asymptomatic [18]. The pathophysiology of this condition is poorly understood. Multiple mechanisms have been implicated, including occult or overt rectal prolapse, dyssynergia, rectal mucosal intussusception, rectal hypersensitivity with a persistent feeling of a need to defecate, and reduced mucosal blood flow [19].

The diagnosis of SRUS is based on the patient’s clinical history combined with endoscopy and histopathology findings. Endoscopically, the lesions may vary in appearance. Shallow ulcerations on hyperemic surrounding mucosa located on the anterior wall is the most common finding [17]. Lesions vary in size, although most are 1 to 1.5 cm in diameter [17] and rarely involve more than half the circumference of the rectal wall. Polypoid lesions occur in approximately 25% of patients with SRUS, and multiple lesions occur in 30% [17].

Obtaining specimens for histology is an important step in the evaluation of SRUS. The differential diagnosis includes Crohn’s disease, ulcerative colitis, ischemic colitis, and malignancy. The typical histologic findings include fibromuscular hyperplasia with smooth muscle infiltration of the lamina propria, thickening of the muscularis mucosa, regenerative changes, and distortion of the crypt architecture [17].

  • Are physiologic or imaging studies helpful in the diagnosis of SRUS?

Two complementary physiologic tests for SRUS are anorectal manometry and defecography. Anorectal manometry often shows evidence of dyssynergia and rectal hypersensitivity in patients with SRUS [20,21]. Hyper-sensitivity may produce a sensation of incomplete evacuation, which in turn results in excessive straining. Defecography may reveal rectal mucosal intussusception or overt rectal prolapse. The patient in this case had evidence of rectal hypersensitivity on anorectal manometry along with excessive perineal descent on defecography.

  • What are treatment options for SRUS?

Treatment of SRUS is not standardized. The options include topical medical therapy, biofeedback, and surgery. Uncontrolled studies have suggested that 5-aminosalicylic acid enema [22], sucralfate enema [23], steroid enema [24], and fibrin glue [25] may improve symptoms. Patients who fail topical therapy and have evidence of dyssynergia on anorectal manometry should receive biofeedback therapy. A case-control study of biofeedback involving 11 patients with refractory SRUS and 15 healthy controls showed improvement in anorectal function, including dyssynergia [21]. At follow-up endoscopy, 36% had complete mucosal healing and more than 50% showed partial healing. In a study involving 16 patients with SRUS and 26 healthy controls, Jarrett et al [26] showed that 75% of patients who underwent biofeedback therapy had improved and 31% had ulcer resolution. Surgical therapy should be considered in rare patients who are refractory to medical therapy. The Delorme procedure is commonly performed with a success rate of 42% to 100% [27].

The case patient underwent biofeedback therapy, and after 5 sessions had complete healing of the lesion and resolution of rectal bleeding and bowel symptoms.

Case Study 3

Initial Presentation and History

A 75-year-old woman is referred to a gastroenterologist with complaints of incomplete stool evacuation and intermittent fecal seepage. She passes stools daily but sits on the toilet for 15 to 20 minutes, and after straining will pass only a small amount of stool. She describes stools as type 4 on the Bristol scale with no blood or mucus. One to 2 hours after a bowel movement, she experiences some wetness in the perineal region and upon checking often notices that a tablespoon full of stool material has leaked out. Sometimes, she will pass another large stool. She denies any leakage of stool while sleeping. Occasionally, she has urgency and leaks stool before reaching the toilet. In the past, she has used digital maneuvers to facilitate stooling. This problem has interfered with shopping, socializing, and taking vacations.

Her past medical history is significant for narcolepsy, hypertension, tubal ligation, appendectomy, and inguinal hernia repair. Obstetric history is significant for 6 vaginal deliveries, 1 requiring episiotomy but no forceps use. Her current medications include estradiol vaginal cream, hydrochlorothiazide, pilocarpine, and amitriptyline 10 mg 3 times daily. She also reports stress urinary incontinence, particularly with sneezing and coughing.

Physical Examination

Physical examination reveals a well-nourished woman with normal vital signs and a normal general examination. Abdominal examination is normal. A rectal examination shows no fissures, but the anocutaneous reflex is absent on the right side. Resting and squeeze sphincter tones are normal, with good perineal descent and normal anal relaxation.

Laboratory Evaluation

Thyroid function tests, complete blood count, and liver biochemistry tests are normal. Results of anorectal manometry are shown in Table 3. Defecography shows a slightly larger resting anorectal angle and an anterior rectocele (Figure 3). Anal endosonography reveals a large sphincter defect in the external anal sphincter (Figure 4).

 

  • What are the mechanisms involved in fecal incontinence?

  • What are the 3 clinical subtypes of fecal incontinence?

Mechanisms and Subtypes

Fecal incontinence is often an unvoiced problem that causes significant social stigma. Approximately 2% of the US population suffers from fecal incontinence [28], with a higher prevalence among women and elderly persons. Several mechanisms are involved in the pathogenesis of fecal incontinence. A common cause is injury to the external or internal anal sphincter, puborectalis muscle, or pudendal nerves, often after obstetric trauma. Hence, a detailed obstetric history including number of vaginal deliveries, use of forceps, tears, and episiotomy is important. Sphincter disruption, most commonly after surgery for hemorrhoid or anal fissure, can result in incontinence. Likewise, reduced rectal compliance causes urgency and fecal incontinence. Impaired rectal sensation results in the accumulation of stool and overflow. Patients rarely have a single cause, with 80% having more than one factor that leads to incontinence [29].

Clinically, fecal incontinence can be classified into 3 categories. Urge incontinence is characterized by the inability to control stool discharge despite active attempts to retain contents. These patients often have disruption or injury to the external anal sphincter. Fecal seepage is the involuntary discharge of less than 2 tablespoons of stool matter without awareness. Seepage can result from impaired rectal evacuation and dyssynergia. Often patients with seepage complain of incomplete evacuation. Passive incontinence refers to the involuntary discharge of stool contents without awareness. These patients often have underlying neuropathy and sphincter weakness [30,31].

  • What is the approach to evaluation and diagnosis?

Evaluation and Diagnosis

Physical examination of patients with fecal incontinence should include a detailed rectal examination, similar to the exam performed in patients who present with constipation. It should include perineal inspection for fissures, fistulae, and skin excoriation. The anocutaneous reflex should be checked along with the resting and squeeze sphincter tone and sphincter relaxation. Further investigations should focus on determining the underlying mechanism in order to facilitate treatment.

Endoscopic investigation should be performed to exclude mucosal disease or malignancy. Anorectal manometry provides objective information regarding resting and squeeze anal sphincter tone, rectal compliance, rectal sensitivity, and rectoanal reflexes [29]. Some experts believe that anorectal manometry is not needed for diagnosis and emphasize the importance of rectal examination and history [32]. Proponents of anorectal manometry point out the importance of physiologic data that can be gained and how it may direct therapy. For example, anorectal manometry and sensory testing may reveal weak anal sphincters and impaired rectal sensation. The latter cannot be identified by clinical evaluation alone. These 2 pathophysiologic findings could enable the biofeedback therapist to focus on improving both anal sphincter tone and rectal sensation [33]. Defecography may reveal anterior rectocele, mucosal intussusception, or rectal prolapse. Anal ultrasound provides information on the structural integrity of the external and internal anal sphincters [34]. Ultrasound is widely available and is relatively inexpensive. Endoanal MRI may provide better information regarding the integrity of the external anal sphincter [35].

  • What are the treatment options?

The goal of treatment is to restore continence and quality of life. General considerations include stool bulking agents such as fiber supplements. Antidiarrheal agents, such as loperamide and diphenoxylate/atropine, are useful as they can decrease stool volume and increase and prolong sphincter pressure and colonic transit time [36,37]. Patients with diarrhea and functional incontinence may benefit from treatment with cholestyramine [38]. Biofeedback therapy improves sphincter tone and rectal sensation [39]. The number of biofeedback sessions is titrated to the patient’s needs, but often 6 sessions are required [40]. Generally, a 70% success rate has been described. Table 4 summarizes recent evidence supporting the use of biofeedback in the treatment of fecal incontinence [41–46].

Surgery for incontinence should be reserved for patients who have failed aggressive conservative management and biofeedback therapy. Overlapping sphincteroplasty is the most common surgery performed for fecal incontinence, with a success rate between 35% and 70% [47,48]. Creation of a neosphincter via dynamic graciloplasty or artificial sphincter has been tried in patients with an irreversibly damaged anal sphincter, but the success rate is low and the complication rate is high [49].

Sacral nerve stimulation (SNS) involves inserting electrodes in the lower back and connecting them to a pulse generator that produces pulses of electricity that innervate the nerves controlling the anal sphincters. Two double-blind crossover studies have reported a beneficial effect of SNS in fecal incontinence [50,51]. In 19 patients who preferred the periods when the stimulator was turned on, the median number of fecal incontinence episodes per week decreased from 1.7 to 0.7, and in the 5 patients who preferred the off period, the median number of fecal incontinence episodes per week increased from 1.7 to 3.7. SNS is now approved by FDA and insurance payers. Recently, hyaluronic acid/dextranomer injection (Solesta, Salix Pharmaceuticals, Raleigh, NC) has also been approved by FDA and has been shown to improve incontinence. A randomized controlled trial showed a 52% response rate to hyaluronic acid/dextranomer compared to a 31% response with placebo [52].

Conclusion

The 3 cases presented illustrate the complexities of several common anorectal disorders. A definitive diagnosis can be established in patients with defecation disorders through systematic evaluations and physiologic and imaging studies. Diagnosis in turn can pave the way for appropriate medical, behavioral, or surgical treatment. If facilities for appropriate testing are unavailable, it is important to refer these patients to appropriate specialists instead of embarking on empirical therapies which may prove futile. Treatment is often possible, and in a majority of patients their symptoms can be ameliorated.

Corresponding author: Satish S.C. Rao, MD, PhD, Section of Gastroenterology and Hepatology, Medical College of Georgia, Georgia Regents University, BB R2540, 1120 15th St., Augusta, GA 30912.

Funding/support: Portions of this work were supported by National Institutes of Health grant RO1 DK 57100-05.

From the Digestive Health Center, Medical College of Georgia, Georgia Regents University, Augusta, GA

 

Defecation is a coordinated process that involves generation of sufficient propulsive forces in the abdomen and rectum together with relaxation of the puborectalis and external anal sphincter. Likewise, continence involves conscious retention of bowel contents until stool or gas can be voluntarily eliminated in an appropriate fashion. A failure of these processes leads to altered bowel function and disorders of defecation that are commonly encountered in clinical practice. They include a diverse group of maladies that result in altered defecation. Among them are functional disorders, such as dyssynergic defecation, and mechanical/structural disorders, such as rectocele, solitary rectal ulcer syndrome (SRUS), excessive perineal descent, and rectal prolapse. This article discusses 3 cases that illustrate the clinical features and management approaches to dyssynergic defecation, SRUS, and fecal incontinence.

Case Study 1

Presentation and History

A 26-year-old white woman with a 10-year history of constipation presents to a gastroenterologist after referral from her primary care physician. She reports spontaneous bowel movements once every 2 weeks, and often she has to induce stools by using enemas or suppositories. Stooling became progressively more difficult for her during her teenage years, with infrequent bowel movements and hard stools (type 1–2 on Bristol stool scale). She also reports having to strain excessively during bowel movements, and on average she spends 30 minutes in the bathroom. She denies experiencing any perianal pain or bleeding or using manual maneuvers to defecate, but she often feels a sense of incomplete evacuation. She also describes intermittent abdominal pain and bloating.

She has tried several over-the-counter laxatives, includ-ing milk of magnesia, senna, and magnesium citrate. Most recently, she tried lubiprostone and polyethylene glycol without improvement. Her past medical history is significant for endometriosis, exploratory laparotomy, and 1 vaginal delivery. There is no family history of colorectal cancer or inflammatory bowel disease. She works as a truck driver and does not use alcohol, illicit drugs, or tobacco. There is no history of physical or sexual abuse. Her current medications include lubiprostone 24 µg twice daily, polyethylene glycol 17 g twice daily, and a birth control pill.

Physical Examination

On physical examination, the patient appears healthy without any distress. Her body mass index is 26 kg/m2, and vital signs are normal. General examination is normal. Abdomen is flat, and bowel sounds are normal. Mild tenderness is noted in both lower quadrants. Rectal examination reveals normal anal skin folds. Digital exam-ination reveals a normal resting tone with pellet-like stool that is heme-negative. When asked to attempt defecation, she shows poor perineal descent and paradoxical contraction of the anal sphincter.

Laboratory Evaluation

Laboratory testing reveals normal levels of thyrotropin and thyroxine, no anemia on complete blood count, and normal levels of calcium, glucose, and electrolytes.

  • What are the possible causes for this patient’s altered bowel habits?

  • What is the approach to physical examination in patients with constipation?

Causes of Constipation

Constipation is a common digestive disorder, affecting up to 20% of the world’s population [1]. Primary or idiopathic constipation consists of 3 common overlapping subtypes: slow-transit constipation, dyssynergic defecation, and constipation-predominant irritable bowel syndrome. Slow-transit constipation involves the slow movement of stool through the colon. This is usually seen on a colonic transit study or with wireless motility capsule study. Dyssynergia in general is caused by functional outlet obstruction with or without normal colonic transit. Patients with dyssynergia often complain of incomplete evacuation, excessive straining, bloating, and blockage [2]. Often patients with dyssynergia resort to manual disimpaction/vaginal splinting and/or abdominal pressure to facilitate bowel movements. Secondary constipation may result from metabolic disorders (eg, hypercalcemia and hypokalemia, disorders associated with renal failure, hypothyroidism, and diabetes) as well as medications, including narcotics, anticholinergics, and antidepressants.

Rectal Examination

Physical examination in patients with constipation should include a detailed rectal examination. The perianal skin should be inspected closely for fissures, fistulae, and skin excoriation. The anocutaneous reflex should be checked along with resting and squeeze anal tone. A study by Rao et al[3] showed that rectal examination could identify 76% of patients with dyssynergia. The sensitivity and positive predictive value for diagnosing dyssynergia with digital rectal examination was 81% and 99%, respectively, making it a good screening test for dyssynergia [3].

  • When is colonoscopy indicated in the workup of constipation?

  • What imaging studies may be useful?

Colonoscopy

Colonoscopic evaluation is only indicated in patients with alarming features such as rectal bleeding, weight loss, unex-plained abdominal pain, palpable mass in the abdomen or rectum, persistent and unexplained anal/rectal pain, or anemia, as well as in patients over age 50 years [4].

Colonic Transit Study

Two imaging studies can be useful in the evaluation of a patient with constipation: colonic transit study and defeco-graphy. A colonic transit study provides useful information regarding the rate at which stool travels through the colon. This test is performed by administering one capsule (Sitzmarks, Konsyl Pharmaceuticals, Fort Worth, TX) containing radiopaque markers. A plain radiograph of the abdomen is obtained on day 6 (120 hr after ingestion of capsule). A transit study is considered abnormal if more than 20% of markers (> 5) are present on a plain radiograph of the abdomen. Approximately two-thirds of patients with dyssynergia have an abnormal colonic transit study, with retention of markers either in the rectosigmoid region or throughout the colon [5]. Wireless motility capsule is a newer test that is comprised of ingesting a capsule and wearing a recorder for up to 5 days. This test measures regional transit (ie, gastric emptying, colonic transit time, and whole gut transit time), is standardized and validated, and avoids use of radiation [6].

Defecography

Defecography is conducted by instilling a barium paste in the rectum and monitoring evacuation of the barium radiologically. It can reveal poor activation of the levator ani muscles, prolonged retention of the barium, inability to expel the barium, absence of a striping wave, rectal mucosal intussusception, rectocele, abnormal perineal descent, or rectal prolapse [5]. Although abnormalities are frequently found on defecography, they may not translate into clinical dysfunction. In one study, 77% of women with complaints of defecation disorders had abnormalities on defecography, but there was no relationship between the abnormalities and the patients’ symptoms [7]. Hence, defecography is not recommended unless there is clinical suspicion of prolapse or excessive descent. Endoanal and dynamic pelvic magnetic resonance imaging (MRI) can evaluate global pelvic floor anatomy in dynamic function [8]. Dynamic MRI in the seated position provides the most physiologic approach.

  • What testing is needed to make a diagnosis of dyssynergic defecation?

Both an abnormal balloon expulsion test and an abnormal pattern of defecation on anal rectal manometry are required to diagnose dyssynergic defecation [9]. Anorectal manometry provides information regarding rectal and anal pressures at rest and during maneuvers of simulated defecation as well as information on rectal sensation, rectoanal reflexes, and compliance [2,10]. There are 4 patterns of dyssynergia found on anorectal manometry: type 1, normal push effort with paradoxical contraction of the anal sphincter; type 2, poor push effort with paradoxical contraction of the anal sphincter; type 3, normal push effort with incomplete or absent relaxation of the anal sphincter; and type 4, poor push with incomplete anal relaxation. The balloon expulsion test should be included in the work-up of dyssynergia.

Normal subjects can expel a 50-mL water-filled balloon in less than 1 minute. Although normal patients can show a dyssynergic pattern in the left lateral decubitus position, when seated on a commode and with a sensation of stooling most exhibit a normal pattern of defecation [9].

Diagnosis

      The case patient undergoes a colonic transit study and has several retained markers distributed primarily in the left colon (Figure 1). Defecography shows incomplete relaxation of the puborectalis, with only 10% of the rectal contents expelled during the study. The results of anorectal manometry and the balloon expulsion test reveal normal sphincter pressures but evidence for an abnormal defecation pattern or dyssynergia: low defecation index (calculated by dividing the rectal pressure by the anal residual pressure during straining) and prolonged balloon expulsion time (Table 1). Based on these findings, a diagnosis of dyssynergic defecation is made.

 

  • What treatment options are available for dyssynergia?

 

The treatment of patients with dyssynergic defecation consists of standard therapies for constipation, including diet, laxatives, and timed toileting. Medical therapy includes laxatives, polyethylene glycol, and lubiprostone. 

Biofeedback therapy is useful and efficacious. Its purpose is to restore a normal pattern of defecation by using an instrument-based education program. The primary goals are to correct dyssynergia and improve rectal sensory perception. Diaphragmatic muscle training, simulated defecation, and manometry- or electromyography-guided anal sphincter and pelvic floor relaxation techniques will improve symptoms in more than 70% of patients [11–13]. Table 2 summarizes data from recent published trials on the efficacy of biofeedback [11–14]. Other avenues of treatment include botulinum toxin injection and surgical approaches that are generally ineffective [15,16].

Case Study 2

Initial Presentation and History

A 39-year-old woman presents with a 5-year history of intermittent bright red blood with stooling. Most often, she notices blood on the toilet paper or when wiping and rarely in the commode. She reports having experienced difficulty with bowel movements since her teens. She does not have a daily urge but strains up to 30 minutes to pass stool that is hard in consistency (type 1–2 on the Bristol stool scale). Over the past year, she has started using fingers to remove stool.

The patient reports bloating and abdominal discomfort that is improved with stooling. Her weight has been stable. Current medications include polyethylene glycol 17 g twice daily, sodium docusate 100 mg twice daily, iron sulfate 325 mg 3 times daily, and a birth control pill. Her past medical history is significant for iron deficiency anemia. Family history is notable for her mother and sister with similar “bowel troubles,” but no family history of inflammatory bowel disease or colorectal cancer. She is a salesperson and has been married for 7 years. She does not use tobacco or alcohol. As a child, she was sexually abused. She did not receive any formal counseling for the abuse. Review of systems is negative.

Physical Examination

General and neurologic examinations are normal. The abdomen is mildly distended, bowel sounds are normal, there is mild tenderness, and stool is palpable in the left lower quadrant. Rectal examination reveals normal anal skin with no fissures, intact anocutaneous reflex, and hard stool in the rectal vault that is guaiac-positive. The resting anal sphincter tone is elevated, and when asked to attempt defecation, there is excessive perineal descent and rectal mucosal intussusception with paradoxical anal contraction.

Laboratory Evaluation and Endoscopy

Laboratory testing reveals normal hemoglobin, white blood cell count, mean corpuscular volume, and electrolytes. Endoscopy is performed and reveals an irregular solitary ulcer base covered in fibrinous exudates in the rectum (Figure 2).

  •  What is SRUS and how is it diagnosed?

Evaluation and Diagnosis

SRUS is characterized by single or multiple ulcerations of the rectal mucosa along with distinct pathologic changes [17]. The term solitary rectal ulcer is a misnomer because many patients have more than 1 lesion, and it is not always an ulcer. Patients with SRUS present with several symptoms, but the most common is passage of blood or mucus, and up to 26% may be asymptomatic [18]. The pathophysiology of this condition is poorly understood. Multiple mechanisms have been implicated, including occult or overt rectal prolapse, dyssynergia, rectal mucosal intussusception, rectal hypersensitivity with a persistent feeling of a need to defecate, and reduced mucosal blood flow [19].

The diagnosis of SRUS is based on the patient’s clinical history combined with endoscopy and histopathology findings. Endoscopically, the lesions may vary in appearance. Shallow ulcerations on hyperemic surrounding mucosa located on the anterior wall is the most common finding [17]. Lesions vary in size, although most are 1 to 1.5 cm in diameter [17] and rarely involve more than half the circumference of the rectal wall. Polypoid lesions occur in approximately 25% of patients with SRUS, and multiple lesions occur in 30% [17].

Obtaining specimens for histology is an important step in the evaluation of SRUS. The differential diagnosis includes Crohn’s disease, ulcerative colitis, ischemic colitis, and malignancy. The typical histologic findings include fibromuscular hyperplasia with smooth muscle infiltration of the lamina propria, thickening of the muscularis mucosa, regenerative changes, and distortion of the crypt architecture [17].

  • Are physiologic or imaging studies helpful in the diagnosis of SRUS?

Two complementary physiologic tests for SRUS are anorectal manometry and defecography. Anorectal manometry often shows evidence of dyssynergia and rectal hypersensitivity in patients with SRUS [20,21]. Hyper-sensitivity may produce a sensation of incomplete evacuation, which in turn results in excessive straining. Defecography may reveal rectal mucosal intussusception or overt rectal prolapse. The patient in this case had evidence of rectal hypersensitivity on anorectal manometry along with excessive perineal descent on defecography.

  • What are treatment options for SRUS?

Treatment of SRUS is not standardized. The options include topical medical therapy, biofeedback, and surgery. Uncontrolled studies have suggested that 5-aminosalicylic acid enema [22], sucralfate enema [23], steroid enema [24], and fibrin glue [25] may improve symptoms. Patients who fail topical therapy and have evidence of dyssynergia on anorectal manometry should receive biofeedback therapy. A case-control study of biofeedback involving 11 patients with refractory SRUS and 15 healthy controls showed improvement in anorectal function, including dyssynergia [21]. At follow-up endoscopy, 36% had complete mucosal healing and more than 50% showed partial healing. In a study involving 16 patients with SRUS and 26 healthy controls, Jarrett et al [26] showed that 75% of patients who underwent biofeedback therapy had improved and 31% had ulcer resolution. Surgical therapy should be considered in rare patients who are refractory to medical therapy. The Delorme procedure is commonly performed with a success rate of 42% to 100% [27].

The case patient underwent biofeedback therapy, and after 5 sessions had complete healing of the lesion and resolution of rectal bleeding and bowel symptoms.

Case Study 3

Initial Presentation and History

A 75-year-old woman is referred to a gastroenterologist with complaints of incomplete stool evacuation and intermittent fecal seepage. She passes stools daily but sits on the toilet for 15 to 20 minutes, and after straining will pass only a small amount of stool. She describes stools as type 4 on the Bristol scale with no blood or mucus. One to 2 hours after a bowel movement, she experiences some wetness in the perineal region and upon checking often notices that a tablespoon full of stool material has leaked out. Sometimes, she will pass another large stool. She denies any leakage of stool while sleeping. Occasionally, she has urgency and leaks stool before reaching the toilet. In the past, she has used digital maneuvers to facilitate stooling. This problem has interfered with shopping, socializing, and taking vacations.

Her past medical history is significant for narcolepsy, hypertension, tubal ligation, appendectomy, and inguinal hernia repair. Obstetric history is significant for 6 vaginal deliveries, 1 requiring episiotomy but no forceps use. Her current medications include estradiol vaginal cream, hydrochlorothiazide, pilocarpine, and amitriptyline 10 mg 3 times daily. She also reports stress urinary incontinence, particularly with sneezing and coughing.

Physical Examination

Physical examination reveals a well-nourished woman with normal vital signs and a normal general examination. Abdominal examination is normal. A rectal examination shows no fissures, but the anocutaneous reflex is absent on the right side. Resting and squeeze sphincter tones are normal, with good perineal descent and normal anal relaxation.

Laboratory Evaluation

Thyroid function tests, complete blood count, and liver biochemistry tests are normal. Results of anorectal manometry are shown in Table 3. Defecography shows a slightly larger resting anorectal angle and an anterior rectocele (Figure 3). Anal endosonography reveals a large sphincter defect in the external anal sphincter (Figure 4).

 

  • What are the mechanisms involved in fecal incontinence?

  • What are the 3 clinical subtypes of fecal incontinence?

Mechanisms and Subtypes

Fecal incontinence is often an unvoiced problem that causes significant social stigma. Approximately 2% of the US population suffers from fecal incontinence [28], with a higher prevalence among women and elderly persons. Several mechanisms are involved in the pathogenesis of fecal incontinence. A common cause is injury to the external or internal anal sphincter, puborectalis muscle, or pudendal nerves, often after obstetric trauma. Hence, a detailed obstetric history including number of vaginal deliveries, use of forceps, tears, and episiotomy is important. Sphincter disruption, most commonly after surgery for hemorrhoid or anal fissure, can result in incontinence. Likewise, reduced rectal compliance causes urgency and fecal incontinence. Impaired rectal sensation results in the accumulation of stool and overflow. Patients rarely have a single cause, with 80% having more than one factor that leads to incontinence [29].

Clinically, fecal incontinence can be classified into 3 categories. Urge incontinence is characterized by the inability to control stool discharge despite active attempts to retain contents. These patients often have disruption or injury to the external anal sphincter. Fecal seepage is the involuntary discharge of less than 2 tablespoons of stool matter without awareness. Seepage can result from impaired rectal evacuation and dyssynergia. Often patients with seepage complain of incomplete evacuation. Passive incontinence refers to the involuntary discharge of stool contents without awareness. These patients often have underlying neuropathy and sphincter weakness [30,31].

  • What is the approach to evaluation and diagnosis?

Evaluation and Diagnosis

Physical examination of patients with fecal incontinence should include a detailed rectal examination, similar to the exam performed in patients who present with constipation. It should include perineal inspection for fissures, fistulae, and skin excoriation. The anocutaneous reflex should be checked along with the resting and squeeze sphincter tone and sphincter relaxation. Further investigations should focus on determining the underlying mechanism in order to facilitate treatment.

Endoscopic investigation should be performed to exclude mucosal disease or malignancy. Anorectal manometry provides objective information regarding resting and squeeze anal sphincter tone, rectal compliance, rectal sensitivity, and rectoanal reflexes [29]. Some experts believe that anorectal manometry is not needed for diagnosis and emphasize the importance of rectal examination and history [32]. Proponents of anorectal manometry point out the importance of physiologic data that can be gained and how it may direct therapy. For example, anorectal manometry and sensory testing may reveal weak anal sphincters and impaired rectal sensation. The latter cannot be identified by clinical evaluation alone. These 2 pathophysiologic findings could enable the biofeedback therapist to focus on improving both anal sphincter tone and rectal sensation [33]. Defecography may reveal anterior rectocele, mucosal intussusception, or rectal prolapse. Anal ultrasound provides information on the structural integrity of the external and internal anal sphincters [34]. Ultrasound is widely available and is relatively inexpensive. Endoanal MRI may provide better information regarding the integrity of the external anal sphincter [35].

  • What are the treatment options?

The goal of treatment is to restore continence and quality of life. General considerations include stool bulking agents such as fiber supplements. Antidiarrheal agents, such as loperamide and diphenoxylate/atropine, are useful as they can decrease stool volume and increase and prolong sphincter pressure and colonic transit time [36,37]. Patients with diarrhea and functional incontinence may benefit from treatment with cholestyramine [38]. Biofeedback therapy improves sphincter tone and rectal sensation [39]. The number of biofeedback sessions is titrated to the patient’s needs, but often 6 sessions are required [40]. Generally, a 70% success rate has been described. Table 4 summarizes recent evidence supporting the use of biofeedback in the treatment of fecal incontinence [41–46].

Surgery for incontinence should be reserved for patients who have failed aggressive conservative management and biofeedback therapy. Overlapping sphincteroplasty is the most common surgery performed for fecal incontinence, with a success rate between 35% and 70% [47,48]. Creation of a neosphincter via dynamic graciloplasty or artificial sphincter has been tried in patients with an irreversibly damaged anal sphincter, but the success rate is low and the complication rate is high [49].

Sacral nerve stimulation (SNS) involves inserting electrodes in the lower back and connecting them to a pulse generator that produces pulses of electricity that innervate the nerves controlling the anal sphincters. Two double-blind crossover studies have reported a beneficial effect of SNS in fecal incontinence [50,51]. In 19 patients who preferred the periods when the stimulator was turned on, the median number of fecal incontinence episodes per week decreased from 1.7 to 0.7, and in the 5 patients who preferred the off period, the median number of fecal incontinence episodes per week increased from 1.7 to 3.7. SNS is now approved by FDA and insurance payers. Recently, hyaluronic acid/dextranomer injection (Solesta, Salix Pharmaceuticals, Raleigh, NC) has also been approved by FDA and has been shown to improve incontinence. A randomized controlled trial showed a 52% response rate to hyaluronic acid/dextranomer compared to a 31% response with placebo [52].

Conclusion

The 3 cases presented illustrate the complexities of several common anorectal disorders. A definitive diagnosis can be established in patients with defecation disorders through systematic evaluations and physiologic and imaging studies. Diagnosis in turn can pave the way for appropriate medical, behavioral, or surgical treatment. If facilities for appropriate testing are unavailable, it is important to refer these patients to appropriate specialists instead of embarking on empirical therapies which may prove futile. Treatment is often possible, and in a majority of patients their symptoms can be ameliorated.

Corresponding author: Satish S.C. Rao, MD, PhD, Section of Gastroenterology and Hepatology, Medical College of Georgia, Georgia Regents University, BB R2540, 1120 15th St., Augusta, GA 30912.

Funding/support: Portions of this work were supported by National Institutes of Health grant RO1 DK 57100-05.

References

1. Higgins PD, Johanson JF. Epidemiology of constipation in North America: a systematic review. Am J Gastroenterol 2004;99:750–9.

2. Rao SS, Mudipalli RS, Stessman M, Zimmerman B. Investigation of the utility of colorectal function tests and Rome II criteria in dyssynergic defecation (Anismus). Neurogastroenterol Motil 2004;16:580–96.

3. Rao SS, Tantiphlachiva K, Rao PS, et al. How useful is digital rectal examination in the assessment of patients with dyssynergia? Am J Gastroenterol 2007;S268:419.

4. Pepin C, Ladabuam U. The yield of lower endoscopy in patients with constipation: survey of a university hospital, public county hospital, and a Veterans Administration medical center. Gastrointest Endosc 2002;56:325–32.

5. Rao SS. Constipation: evaluation and treatment. Gastroenterol Clin North Am 2003;32:659–83.

6. Rao SS, Camilleri M, Hasler WL, et al. Evaluation of  gastrointestinal transit in clinical practice: position paper of the  American and European Neurogastroenterology and Motility Societies. Neurogastroenterol Motil; 2011; 23: 8-23.

7. Savoye-Collet C, Savoye G, Koning E, et al. Defecography in symptomatic older women living at home. Age Ageing 2003;32:347–50.

8. Bolog N, Weishaupt D. Dynamic MR imaging of outlet obstruction. Rom J Gastroenterol 2005;14:293–302.

9. Rao SS, Kavlock R, Rao S. Influence of body position and stool characteristics on defecation in humans. Am J Gastroenterol 2006;101:2790–6.

10. Karlbom U, Lundin E, Graf W, Pahlman L. Anorectal physiology in relation to clinical subgroups of patients with severe constipation. Colorectal Dis 2004;6:343–9.

11. Chiarioni G, Whitehead WE, Pezza V, et al. Biofeedback is superior to laxatives for normal transit constipation due to pelvic floor dyssynergia. Gastroenterology 2006;130:657–64.

12. Rao SS, Seaton K, Miller M, et al. Randomized controlled trial of biofeedback, sham feedback, and standard therapy for dyssynergic defecation. Clin Gastroenterol Hepatol 2007;5:331–8.

13. Chiarioni G, Saladini L, Whitehead W. Biofeedback benefits only patients with outlet dysfunction, not patients with isolated slow transit constipation. Gastroenterology 2005;129:86–97.

14. Heymen S, Scarlett Y, Jones K, et al. Randomized controlled trial shows biofeedback to be superior to alternate treatments for patients with pelvic floor dyssynergia-type defecation [abstract]. Am J Gastroenterol 2005;100:S335.

15. Hallan RI, Williams NS, Melling J, et al. Treatment of anismus in intractable constipation with botulinum A toxin. Lancet 1988;2:714–7.

16. Boccasanta P, Venturi M, Stuto A, et al. Stapled transanal rectal resection for outlet obstruction: a prospective, multicenter trial. Dis Colon Rectum 2004;47:1285–97.

17. Sharara AI, Azar C, Amr SS, et al. Solitary rectal ulcer syndrome: endoscopic spectrum and review of the literature. Gastrointest Endosc 2005;62:755–62.

18. Tjandra JJ, Fazio VW, Church JM, et al. Clinical conundrum of solitary rectal ulcer. Dis Colon Rectum 1992;35:227–34.

19. Felt-Bersma RJ, Cuesta A. Rectal prolapse, rectal intussusception, rectocele, and solitary rectal ulcer syndrome. Gastroenterol Clin North Am 2001;30:199–222.

20. Vaizey CJ, van den Bogaerde JB, Emmanuel AV, et al. Solitary rectal ulcer syndrome. Br J Surg 1998;85:1617–23.

21. Rao SS, Ozturk R, De Ocampo S, Stessman M. Pathophysiology and role of biofeedback therapy in solitary rectal ulcer syndrome. Am J Gastroenterol 2006;101:613–8.

22. Kumar M, Puri AS, Srivastava R, Yachha SK. Solitary rectal ulcer syndrome in a child treated with local sulfasalazine. Indian Pediatr 1994;31:1553–5.

23. Zargar SA, Khuroo MS, Mahajan R. Sucralfate retention enemas in solitary rectal ulcer. Dis Colon Rectum 1991;34:455–7.

24. Bishop PR, Nowicki MJ. Nonsurgical therapy for solitary rectal ulcer syndrome. Curr Treat Options Gastroenterol 2002;5:215–23.

25. Ederle A, Bulighin G, Orlandin PG, Pilati S. Endoscopic application of human fibrin sealant in the treatment of solitary rectal ulcer syndrome [letter]. Endoscopy 1992;24:736–7.

26. Jarrett ME, Emmanuel AV, Vaizey CJ, Kamm MA. Behavioural therapy (biofeedback) for solitary rectal ulcer syndrome improves symptoms and mucosal blood flow. Gut 2004;53:368–70.

27. Tweedie DJ, Varma JS. Long-term outcome of laparoscopic mesh rectopexy for solitary rectal ulcer syndrome. Colorectal Dis 2005;7:151–5.

28. Rao SS. Diagnosis and management of fecal incontinence. American College of Gastroenterology Practice Parameters Committee. Am J Gastrenterol 2004;99:1585–604.

29. Rao SS, Patel RS. How useful are manometric tests of anorectal function in the management of defecation disorders? Am J Gastroenterol 1997;92:469–75.

30. Rao SS. Pathophysiology of adult fecal incontinence. Gastroenterology 2004;126(1 Suppl 1):S14–22.

31. Deutekom M, Dobben AC, Terra MP, et al. Clinical presentation of fecal incontinence and anorectal function: what is the relationship? Am J Gastroenterol 2007;102:351–61.

32. Wald A. Con: anorectal manometry and imaging are not necessary in patients with fecal incontinence. Am J Gastroenterol 2006;101:2681–3.

33. Bharucha AE. Pro: anorectal testing is useful in fecal incontinence. Am J Gastroenterol 2006;101:2679–81.

34. Tuteja AK, Rao SS. Review article: recent trends in diagnosis and treatment of faecal incontinence. Aliment Pharmacol Ther 2004;19:829–40.

35. Terra MP, Beets-Tan RG, van der Hulst VP, et al. MRI in evaluating atrophy of the external anal sphincter in patients with fecal incontinence. AJR Am J Roentgenol 2006;187:991–9.

36. Sun WM, Read NW, Verlinden M. Effects of loperamide oxide on gastrointestinal transit time and anorectal function in patients with chronic diarrhoea and faecal incontinence. Scan J Gastroenterol 1997;32:34–8.

37. Harford WV, Krejs GJ, Santa Ana CA, Fordtran JS. Acute effect of diphenoxylate with atropine (Lomotil) in patients with chronic diarrhea and fecal incontinence. Gastroenterol 1980;78:440–3.

38. Remes-Troche JM, Ozturk R, Philips C, et al. Cholesteryramine—a useful adjunct for the treatment of patients with fecal incontinence. Int J Colorectal Dis 2008;23:189–99.

39. Rao SS. The technical aspects of biofeedback therapy for defecation disorders. Gastroenterologist 1998;6:96–103.

40. Rao SS, Welcher KD, Happel J. Can biofeedback therapy improve anorectal function in fecal incontinence? Am J Gastroenterol 1966;91:2360–6.

41. Byrne CM, Solomon MJ, Young JM, et al. Biofeedback for fecal incontinence: short-term outcomes of 513 consecutive patients and predictors of successful treatment. Dis Colon Rectum 2007;50:417–27.

42. Norton C, Chelvanayagam S, Wilson-Barnett J, et al. Randomized controlled trial of biofeedback for fecal incontinence. Gastroenteroloy 2003;125:1320–9.

43. Heymen S, Whitehead W. EMG biofeedback vs. Kegel exercise for the treatment of fecal incontinence. Gastroenterology 2007;132:A–83.

44. Pager CK, Solomon MJ, Rex J, Roberts RA. Long-term outcomes of pelvic floor exercise and biofeedback treatment for patients with fecal incontinence. Dis Colon Rectum 2002;45:997–1003.

45. Ozturk R, Niazi S, Stessman M, Rao SS. Long-term and objective changes of anorectal function after biofeedback therapy for faecal incontinence. Aliment Pharmacol Ther 2004;20:667–74.

46. Byrne CM, Solomon MJ, Rex J, et al. Telephone vs. face-to-face biofeedback for fecal incontinence: comparison of two techniques in 239 patients. Dis Colon Rectum 2005;48:2281–8.

47. Zorcolo L, Covotta L, Bartolo DC. Outcome of anterior sphincter repair for obstetric injury: comparison of early and late results. Dis Colon Rectum 2005;48:524–31.

48. Madoff RD. Surgical treatment options for fecal incontinence. Gastroenterology 2004;126(1 Suppl 1)S48–54.

49. Thornton MJ, Kennedy ML, Lubowski DZ, King DW. Long-term follow-up of dynamic graciloplasty for faecal incontinence. Colorectal Dis 2004;6:470–6.

50. Leroi AM, Parc Y, Lehur PA, et al. Efficacy of sacral nerve stimulation for fecal incontinence: results of a multicenter double-blind crossover study. Ann Surg 2005;242:662–9.

51. Vaizey CJ, Kamm MA, Nicholls RJ. Double-blind crossover study of sacral nerve stimulation for fecal incontinence. Dis Colon Rectum 2000;43:298–302.

52. Graf W, Mellgren A, Matzel KE, et al. . Efficacy of dextranomer in stabilised hyaluronic acid for treatment of faecal incontinence: a randomised, sham-controlled trial. Lancet;2011;377:997–1003.

References

1. Higgins PD, Johanson JF. Epidemiology of constipation in North America: a systematic review. Am J Gastroenterol 2004;99:750–9.

2. Rao SS, Mudipalli RS, Stessman M, Zimmerman B. Investigation of the utility of colorectal function tests and Rome II criteria in dyssynergic defecation (Anismus). Neurogastroenterol Motil 2004;16:580–96.

3. Rao SS, Tantiphlachiva K, Rao PS, et al. How useful is digital rectal examination in the assessment of patients with dyssynergia? Am J Gastroenterol 2007;S268:419.

4. Pepin C, Ladabuam U. The yield of lower endoscopy in patients with constipation: survey of a university hospital, public county hospital, and a Veterans Administration medical center. Gastrointest Endosc 2002;56:325–32.

5. Rao SS. Constipation: evaluation and treatment. Gastroenterol Clin North Am 2003;32:659–83.

6. Rao SS, Camilleri M, Hasler WL, et al. Evaluation of  gastrointestinal transit in clinical practice: position paper of the  American and European Neurogastroenterology and Motility Societies. Neurogastroenterol Motil; 2011; 23: 8-23.

7. Savoye-Collet C, Savoye G, Koning E, et al. Defecography in symptomatic older women living at home. Age Ageing 2003;32:347–50.

8. Bolog N, Weishaupt D. Dynamic MR imaging of outlet obstruction. Rom J Gastroenterol 2005;14:293–302.

9. Rao SS, Kavlock R, Rao S. Influence of body position and stool characteristics on defecation in humans. Am J Gastroenterol 2006;101:2790–6.

10. Karlbom U, Lundin E, Graf W, Pahlman L. Anorectal physiology in relation to clinical subgroups of patients with severe constipation. Colorectal Dis 2004;6:343–9.

11. Chiarioni G, Whitehead WE, Pezza V, et al. Biofeedback is superior to laxatives for normal transit constipation due to pelvic floor dyssynergia. Gastroenterology 2006;130:657–64.

12. Rao SS, Seaton K, Miller M, et al. Randomized controlled trial of biofeedback, sham feedback, and standard therapy for dyssynergic defecation. Clin Gastroenterol Hepatol 2007;5:331–8.

13. Chiarioni G, Saladini L, Whitehead W. Biofeedback benefits only patients with outlet dysfunction, not patients with isolated slow transit constipation. Gastroenterology 2005;129:86–97.

14. Heymen S, Scarlett Y, Jones K, et al. Randomized controlled trial shows biofeedback to be superior to alternate treatments for patients with pelvic floor dyssynergia-type defecation [abstract]. Am J Gastroenterol 2005;100:S335.

15. Hallan RI, Williams NS, Melling J, et al. Treatment of anismus in intractable constipation with botulinum A toxin. Lancet 1988;2:714–7.

16. Boccasanta P, Venturi M, Stuto A, et al. Stapled transanal rectal resection for outlet obstruction: a prospective, multicenter trial. Dis Colon Rectum 2004;47:1285–97.

17. Sharara AI, Azar C, Amr SS, et al. Solitary rectal ulcer syndrome: endoscopic spectrum and review of the literature. Gastrointest Endosc 2005;62:755–62.

18. Tjandra JJ, Fazio VW, Church JM, et al. Clinical conundrum of solitary rectal ulcer. Dis Colon Rectum 1992;35:227–34.

19. Felt-Bersma RJ, Cuesta A. Rectal prolapse, rectal intussusception, rectocele, and solitary rectal ulcer syndrome. Gastroenterol Clin North Am 2001;30:199–222.

20. Vaizey CJ, van den Bogaerde JB, Emmanuel AV, et al. Solitary rectal ulcer syndrome. Br J Surg 1998;85:1617–23.

21. Rao SS, Ozturk R, De Ocampo S, Stessman M. Pathophysiology and role of biofeedback therapy in solitary rectal ulcer syndrome. Am J Gastroenterol 2006;101:613–8.

22. Kumar M, Puri AS, Srivastava R, Yachha SK. Solitary rectal ulcer syndrome in a child treated with local sulfasalazine. Indian Pediatr 1994;31:1553–5.

23. Zargar SA, Khuroo MS, Mahajan R. Sucralfate retention enemas in solitary rectal ulcer. Dis Colon Rectum 1991;34:455–7.

24. Bishop PR, Nowicki MJ. Nonsurgical therapy for solitary rectal ulcer syndrome. Curr Treat Options Gastroenterol 2002;5:215–23.

25. Ederle A, Bulighin G, Orlandin PG, Pilati S. Endoscopic application of human fibrin sealant in the treatment of solitary rectal ulcer syndrome [letter]. Endoscopy 1992;24:736–7.

26. Jarrett ME, Emmanuel AV, Vaizey CJ, Kamm MA. Behavioural therapy (biofeedback) for solitary rectal ulcer syndrome improves symptoms and mucosal blood flow. Gut 2004;53:368–70.

27. Tweedie DJ, Varma JS. Long-term outcome of laparoscopic mesh rectopexy for solitary rectal ulcer syndrome. Colorectal Dis 2005;7:151–5.

28. Rao SS. Diagnosis and management of fecal incontinence. American College of Gastroenterology Practice Parameters Committee. Am J Gastrenterol 2004;99:1585–604.

29. Rao SS, Patel RS. How useful are manometric tests of anorectal function in the management of defecation disorders? Am J Gastroenterol 1997;92:469–75.

30. Rao SS. Pathophysiology of adult fecal incontinence. Gastroenterology 2004;126(1 Suppl 1):S14–22.

31. Deutekom M, Dobben AC, Terra MP, et al. Clinical presentation of fecal incontinence and anorectal function: what is the relationship? Am J Gastroenterol 2007;102:351–61.

32. Wald A. Con: anorectal manometry and imaging are not necessary in patients with fecal incontinence. Am J Gastroenterol 2006;101:2681–3.

33. Bharucha AE. Pro: anorectal testing is useful in fecal incontinence. Am J Gastroenterol 2006;101:2679–81.

34. Tuteja AK, Rao SS. Review article: recent trends in diagnosis and treatment of faecal incontinence. Aliment Pharmacol Ther 2004;19:829–40.

35. Terra MP, Beets-Tan RG, van der Hulst VP, et al. MRI in evaluating atrophy of the external anal sphincter in patients with fecal incontinence. AJR Am J Roentgenol 2006;187:991–9.

36. Sun WM, Read NW, Verlinden M. Effects of loperamide oxide on gastrointestinal transit time and anorectal function in patients with chronic diarrhoea and faecal incontinence. Scan J Gastroenterol 1997;32:34–8.

37. Harford WV, Krejs GJ, Santa Ana CA, Fordtran JS. Acute effect of diphenoxylate with atropine (Lomotil) in patients with chronic diarrhea and fecal incontinence. Gastroenterol 1980;78:440–3.

38. Remes-Troche JM, Ozturk R, Philips C, et al. Cholesteryramine—a useful adjunct for the treatment of patients with fecal incontinence. Int J Colorectal Dis 2008;23:189–99.

39. Rao SS. The technical aspects of biofeedback therapy for defecation disorders. Gastroenterologist 1998;6:96–103.

40. Rao SS, Welcher KD, Happel J. Can biofeedback therapy improve anorectal function in fecal incontinence? Am J Gastroenterol 1966;91:2360–6.

41. Byrne CM, Solomon MJ, Young JM, et al. Biofeedback for fecal incontinence: short-term outcomes of 513 consecutive patients and predictors of successful treatment. Dis Colon Rectum 2007;50:417–27.

42. Norton C, Chelvanayagam S, Wilson-Barnett J, et al. Randomized controlled trial of biofeedback for fecal incontinence. Gastroenteroloy 2003;125:1320–9.

43. Heymen S, Whitehead W. EMG biofeedback vs. Kegel exercise for the treatment of fecal incontinence. Gastroenterology 2007;132:A–83.

44. Pager CK, Solomon MJ, Rex J, Roberts RA. Long-term outcomes of pelvic floor exercise and biofeedback treatment for patients with fecal incontinence. Dis Colon Rectum 2002;45:997–1003.

45. Ozturk R, Niazi S, Stessman M, Rao SS. Long-term and objective changes of anorectal function after biofeedback therapy for faecal incontinence. Aliment Pharmacol Ther 2004;20:667–74.

46. Byrne CM, Solomon MJ, Rex J, et al. Telephone vs. face-to-face biofeedback for fecal incontinence: comparison of two techniques in 239 patients. Dis Colon Rectum 2005;48:2281–8.

47. Zorcolo L, Covotta L, Bartolo DC. Outcome of anterior sphincter repair for obstetric injury: comparison of early and late results. Dis Colon Rectum 2005;48:524–31.

48. Madoff RD. Surgical treatment options for fecal incontinence. Gastroenterology 2004;126(1 Suppl 1)S48–54.

49. Thornton MJ, Kennedy ML, Lubowski DZ, King DW. Long-term follow-up of dynamic graciloplasty for faecal incontinence. Colorectal Dis 2004;6:470–6.

50. Leroi AM, Parc Y, Lehur PA, et al. Efficacy of sacral nerve stimulation for fecal incontinence: results of a multicenter double-blind crossover study. Ann Surg 2005;242:662–9.

51. Vaizey CJ, Kamm MA, Nicholls RJ. Double-blind crossover study of sacral nerve stimulation for fecal incontinence. Dis Colon Rectum 2000;43:298–302.

52. Graf W, Mellgren A, Matzel KE, et al. . Efficacy of dextranomer in stabilised hyaluronic acid for treatment of faecal incontinence: a randomised, sham-controlled trial. Lancet;2011;377:997–1003.

Issue
Journal of Clinical Outcomes Management - OCTOBER 2014, VOL. 21, NO. 10
Issue
Journal of Clinical Outcomes Management - OCTOBER 2014, VOL. 21, NO. 10
Publications
Publications
Topics
Article Type
Display Headline
Defecation Disorders: Diagnosis and Treatment
Display Headline
Defecation Disorders: Diagnosis and Treatment
Sections
Disallow All Ads
Content Gating
No Gating (article Unlocked/Free)
Alternative CME
Disqus Comments
Default